Thursday, September 9, 2010

'Space' Warfare XII: Surface Warfare


Ask, and ye shall (sometimes) receive: A reader emailed asking me to discuss future ground war. This I will take a bit more broadly as warfare fought on habitable, shirtsleeves planets, including sea and air operations. Warfare on non-habitable planets is an ambiguous case, with features of boarding operations in space.

I intended to deal first with the space context. But y'all want grunts, preferably in power armor, supported by coolific armored vehicles and aircraft, with subs and trimaran assault cruisers out to sea. Which brings us to something that has not been tested yet. What happens when post industrial forces fight each other?

We don't know, but we have seen this movie before, in flickery black and white a hundred years ago. Industrial age Western armies had shown how well they could scythe down waves of natives, usually. The general prognosis was that 1900-modern weapons were so accurate and effective that when turned on each other they would pretty much wipe each other out, and quickly.

The Next War would be a come-as-you-are war, settled in months if not weeks by whoever ran out of arms and ammo first, if it weren't won a week earlier by strategy and tactical execution. 1870 was the prelude; 1861-65 merely an example of a semi-modern war fought entirely by blundering amateurs.

It did not work out that way in 1914, so I hesitate to say it would work out that way now, or in 2114. What might happen, in fact, is broadly what happened in 1914: Everyone goes to ground.

The general principle of future surface war, it seems to me, is that if you are caught out in the open you are headed for the celestial choir. This goes for guerillas, it goes for power armor troops, it goes for laser armed tanks, trimaran cruisers, aircraft, and spacecraft in low orbit. Give precision weapons a clear target and they will take it out.

Thirty meter mecha, sad to say, make for very clear targets.

Reconnaissance robotics, on the other hand, will be hard to take out. They can be very small and stealthy, making the Predator look like a B-36. So you should have plenty of scouts, including a robotic fly on the wall of the other side's headquarters. Your intel problem is noise - the more raw intel, the more noise. Any AI good enough to cut through it is an intelligence officer, not a piece of equipment.

A tank backed into the underbrush is still effective, because it is hard to find, and you may only find it when it opens up on you. A tank on the move has a target painted on it. This, I think, is the real advantage of power armor troops: Compared to tanks they are stealthy, and can slip through environments where a tank would draw attention and fire.

I expect power armor to be relatively light. At minimum you want enough to stop small arms fire, shrapnel, and the like. The maximum of useful armor is reached when a hit would kill you anyway, like getting hit by the equivalent of a truck. Against lasers this may mean the point at which you cook inside your armor, not good.

Future war may well be 'slow,' because the mobility of power armor troops is essentially foot mobility, with enhancements like powered roller skates. Mobility is limited behind the front as well, because truck convoys will be conspicuous targets even hundreds of km behind the lines. Logistics too will have to be stealthy.

It is easier to have a Ho Chi Minh Trail in the jungle, so one thing timeless will be the supreme importance of ground and the physical ecosystem. This of course gets interesting on habitable planets other than Earth.

Some kinds of fortifications might remain valid, basically because dirt absorbs a lot of damage points. Yes, there are bunker busters and Thor bolts, but the point is that such big powerful weapons are costly to deploy, carried by vulnerable platforms, and can be engaged by defensive fire. This could be the saving of large naval surface combatants, hard to sink except by massive attack that overwhelms their defenses.

The one way to achieve rapid, heavy movement, whether logistic or an actual assault, is to ramp up the noise level so high that the enemy's sensors are saturated, and nothing (you hope) is in 'plain sight.' If you are right you get blitzkrieg; if wrong you get the Somme.

Large scale surface war may thus have an alternating rhythm - weeks or months of stalking, skulking, and skirmishes along the front, interrupted by episodes of sheer rock & roll, perhaps to cover the fast movement of a truck convoy up to the front, where it will disperse itself and go to ground.

All of this takes place, or doesn't, against the background of nuclear weapons. The constraints on mobility in 'conventional' warfare could make it indecisive enough for the great powers to engage in it without risking a nuclear exchange. As in the 18th century they would be fighting for provinces, not national survival.


Now for the space context. If suitable planets are limited - say, Earth and terraformed Mars or Venus - politically balkanized planets are to be expected, unless you go mid 20th century retro and have the American Empire a Federation. Certainly on Earth itself you can plausibly expect Great Powers, with great power militaries.

In a few-worlds setting, space itself will be off in the background. India is not going to get in a major tussle with Olympus Mons; both have bigger problems much closer to home. And India and China are not going to take their arguments to the asteroid belt, at least not in a big way, because money spent on deep space forces comes out of much more critical surface, air, and Earth orbital forces.

This can have advantages for space-centric settings, because you can let the major Earth powers stalemate each other, keeping them off the deep space chessboard.

In the classic operatic setting of many colony worlds, it could be a different matter. Uniform planets are rightly bashed, and I've bashed them myself. But in such a setting I think politically unified planets will be common, perhaps the norm. In the colonization era everyone can have their own planet, and later on, even if local fissures develop - and they will - any planet that can present a united front enjoys a huge advantage in interstellar power politics.

Or putting it another way, any planet that cannot present a united front is at a huge disadvantage, drawing plot complications like flies. And here we are.


I could make the many and salient arguments for peace, but I know they would fall on deaf ears, so we'll go straight to comments.


The image comes from this futurist blog.

828 comments:

«Oldest   ‹Older   601 – 800 of 828   Newer›   Newest»
Luke said...

Milo:

Are they accurate enough to hit inside the crater made a few seconds previously by the same laser, from huge ranges against a moving target?

For orbital bombardment, we probably are not looking at craters. Rather, you use the lasers as heat rays. With multi-megawatts of power, you can do a decent job against infantry and light armor.

There's no reason not to use thick channels, is there? The total amount of electricity I transmit is more important than where I put it, so any spot size smaller than the size of the target should work fine.

Pretty much. You do need to have the channels small enough that they don't overlap while still fitting in something that is not too big.

That's not bad at all. I think you get similar amounts of power just from standing sunlight.

Also note that most of this power is dumped into the air rather than the target - in my example, all electrical power over 18 W is used to heat the conductive channels.

So how would this beam work (when set to "stun" rather than "kill")?

If set to kill, you need a higher current. This means you either need more ionization of the air to lower the resistance, or a higher voltage to drive more current through the same resistance (or both).

How long do I need to keep it on an enemy?

Modern Tasers knock a person down immediately, and keep most people down for as long as the current is applied. Several seconds of continual tasing results in lack of coordination and fatigue that can last after the current is turned off. Some very tough people have demonstrated the ability to retain some voluntary muscle control even under the effect of a Taser - although they are still significantly impaired as long as the current is still flowing through them.

So, you are probably looking at needing to hold the beam on your target for several seconds in order to capture it. For self defense, merely knocking an assailant down and running may be sufficient.

Kill effects may work in a fraction of a second on little old ladies or overweight guys who eat too much junk food. On the other hand, healthy people who get a lot of exercise might just get right back up after getting a killing shock. Electricity tends to produce unreliable results - occasionally you will kill healthy people, too, and occasionally even people with weak hearts will survive. There is even a weird effect where too much electric current can be less effective - instead of causing ventricular fibrillation it can simply stop the heart cold, allowing the heart to restart itself.

Turbo10k said...

w00t! YEARHH!

I finally managed to read through all that ... thing.

Turbo10k said...

Since there is no way I can reply to even a part of the previous posts, I'll try and put a post for each 'topic' brought up, and in an easy-to-reply to bullet point format.

First one and the easiest:

Submarine-to-Orbit warfare:

-Mini Sub drops are stupid and pointless. First off because mini-subs are as stupid from a military point of view as space fighters are to spacewarships. Smaller, slower (if speed is proportional to propeller size and electricity generation) and highly limited in crew and ammo. You could use small, zippy small subs (using hydrojets?) but they would lose out in stealth and get shot down (sunk?) immediately once they enter one of the bigger, defensive subs.

-For the same mass and logistics load of a minisub, I can drop many more torpedoes. Torpedoes have the advantage over mini-subs in terms of lighter mass, expendability and having no need for stealth, over kinetic strikes as they don't have to go through several mediums at speed, and over lasers as the latter can't hit what it can't see.
Also, orbital-drop torpedoes can use massive active sonar detection (explosively-driven sonar?) to find the sub if the latter decides to move out of the area after a laser strike or makes a submerged ballistic launch.

-Submarine pens are easy to find, or at least easier to detect than anything else on the ground, flying or underwater. You can't even disguise them as civilian ports. Submarines will leave their pens at the start of the war and stay submerged for as long as possible, so crew strain will count.

Turbo10k said...

PS: How many terraformed planets are supposed to have large, deep oceans?

Turbo10k said...

Two!

Troops from SPAAACE!:
Only the orbital drop part)

-In your setting, do you have politically boring, uniform planets where defences are truly global and maps use a single colour? If so, say goodbye to conflict. If not, then you have based your setting on five millenia of human history. It also suggests that the whole planet is not unified in reponse to your invasion, and if any nation from 1 to 199 decides to help you, you have your well-established gap in the global defences without any bombardment required. You will therefore have troops dropped in the middle of the assault without need for magitech/hard to describe and awkward tactics to justify troop presence.

-If above is true for you, this discussion should answer "How can Orbital Drop troops help present ground forces?" rather than "Do we need them at all during the invasion?"

In other words, a bit of political realism and we've solved a huge problem.

-Drop pods seem to have predictable paths during their descent, all while being fragile in this discussion. Why can't we launch aerospaceplane troop transporters that dodge while in the atmosphere then make a graceful, slow landing? Say, we drop a jumbo-jet version of the Space Shuttle, with a large spade shaped heat shield that reduces speed to that manageable by carbon-lined wings. Despite the plasma sheath still present, I think it will have sufficient maneouverability and evasion capability if 1)It is guided by space-based observers 2)Has large flaps that provide some lift at lower speeds.

-If you find it hard to drop small troop transports, how's it going to be for the ridiculous Ships and Other Large Structures from SPAAACE! propositions? Drop them in pieces?

Turbo10k said...

More coming later.

Luke said...

Turbo10k:

An aerospace transport will not be able to maneuver as well as an air-to-space missile. This makes dodging against missiles implausible. Dodging a laser is, of course, impossible, although the plasma sheath from re-entry may provide a certain amount of protection.

Luke said...

Err, let me qualify that by saying that the transport cannot maneuver as well as the missile while in the atmosphere. With aerodynamic control surfaces, you have the square-cube law operating in favor of the smaller body, plus the missile can be almost entirely optimized for maneuverability while the transport has little room for optimization if it is going to transport troops through the aerospace interface. In space, it is a different story, where maneuver is decided by delta-V rather than control surfaces.

Milo said...

"PS: How many terraformed planets are supposed to have large, deep oceans?"

That depends on how much you choose to give them :)

If you're limited to melting only the water naturally present on the planet a priori, without having the resources to hailbombs, then seas will be thin and patchy.

If you can do some hailbombing, however, it's desirable to bring in a lot more. Yes, we're land animals and so you might think that flooding the planet would be a bad idea - but it doesn't work that way. A planet with too little ocean will have wide swaths of desert - land, sure, but not land we'd particularly care to live in. In practice it turns out that raising sea levels will usually do more good than harm in terms of increasing fertile land area. You don't want to flood everything, of course, but ocean covering well over half the planet's surface is likely to be desirable.

Another thing to pay attention to is where you put the oceans. Barring a massive earthmoving project, you don't get to decide this - it depends on the planet's preexisting lowlands. On Mars, in particular, most of the sea will be in the northern hemisphere, and most of the land will be in the southern hemisphere. Moreso in the latter than the former - there are still some significant highlands in the northern hemisphere (Tharsis, Arabia, Elysium), while the southern hemisphere has a few roughly-circular crater seas of significant size (especially Hellas and Argyre). This is not ideal - you want more mixing of land and water. Perhaps flooding it a little higher (higher than the oceans fossil coastline suggests Mars had last time) would allow you to flood Nirgal Valley and connect the Argyre Sea to the ocean, which would allow significantly more water to cut through the southern hemisphere, which would be quite beneficial ecologically.

Luke said...

More thoughts on water on terraformed worlds:

If you are restricting yourself to our solar system, Milo pretty much covered the relevant points. Extra-solar planets could be quite different, with highly variable amounts of water already present. One possibility thought likely by exogeologists is a world ocean, very often one so thick that the pressure at the bottom forces the water into ice. You would get a mantle of ice over a core of rock, all covered with a thin shell of water. In extremes, this gets you ice giants like Uranus and Neptune, but I could imagine an ice world of this sort in the habitable and with a reasonable surface gravity, terraformed to hold an oxygen atmosphere and with floating arcologies dotting the surface.

Planets of diamond and silicon carbide are another wonderfully weird possibility. Since these will be from solar systems that are very oxygen poor, you will have essentially no water to put on them.

Milo said...

Luke:

"One possibility thought likely by exogeologists is a world ocean,"

These aren't likely to be very healthy locations for life, either natural or terraformed - as before, it's a mixture of land and water that promotes life. Land creatures like it best when there's lots of rain, and sea creatures like it best in shallow waters where they have access to both soil and sunlight.

You could build a few floating arcologies, but these have little obvious advantage over domed cities. The point of terraforming is to have some sort of nature outside the immediate human settlements.

If I were tasked with terraforming such a place, I would see the excess water as a problem that needs to be removed - like Venus's atmosphere. Of course, not too much. I would be happy to drain it down to an ocean dotted with islands - that should be enough.


"Planets of diamond and silicon carbide are another wonderfully weird possibility. Since these will be from solar systems that are very oxygen poor, you will have essentially no water to put on them."

Meaning we're unlikely to choose those solar systems for settlement! Unless interstellar travel is so cheap that we can afford to import our water.

I would love to find natural life that arose on such a planet, though. It would be a delightfully exotic ecosphere - most likely methane-based, like Titan.

That said, might it be that there still is enough oxygen to form water, just not to form large amounts of silica? Hydrogen is much more common than silicon, so if your solar system only has a little oxygen, I'd expect most of it would become water before moving on to making silica. And, after all, we don't need a lot of water, as long as it's in the right place - water only constitutes a tiny fraction of Earth's total mass. Then again, even in our solar system, we got plenty of silica despite also still having plenty of molecular hydrogen left...

Thucydides said...

Another possibility is planets formed in a solar system with large amounts of carbon present. The planetscape would be dominated by oceans of tar and other hydrocarbons and the land features might be continents of diamond.

Once again, this isn't the sort of place "we" would consider unless external reasons made the system attractive (being a node of a series of Alderson point tramlines, for example). Non human life forms might have their own reasons to choose such worlds, in which case our military response would be exterminate with extreme prejudice (if they were considered an existential threat) to ignore (if they pose no threat). Limited threats would be subjected to space blockade, as an economy of force measure and to prevent the problem from growing and spreading.

These sorts of worlds would seem to fall under Space warfare rather than surface warfare, given the need to "seize and hold ground" is pretty limited.

Milo said...

Thucydides:

"Non human life forms might have their own reasons to choose such worlds, in which case our military response would be exterminate with extreme prejudice (if they were considered an existential threat) to ignore (if they pose no threat)."

That's a pretty wide range in response!

But them not posing a threat is pretty likely. If two intelligent species evolved for very different kinds of planet, they might well coexist in the same solar system without getting in each other's way, since they aren't in competition for the same planets. If, for example, we were contacted by aliens adapted to several-hundred-degree temperatures that are looking for new living space to terraform, I'd happily tell them they can have Mercury if they share with us the technology they used to travel here :)


"These sorts of worlds would seem to fall under Space warfare rather than surface warfare, given the need to "seize and hold ground" is pretty limited."

Unless there are MacGuffinite mines on the world.

Or... if it's inhabited by multiple factions of alien species that are at war with each other, and humans are for whatever reason allied to one of them. That would give us a reason not to slag it! Although we'd probably still stick to orbital fire support and let the natives do the ground fighting.

Milo said...

Incidentally, war with aliens is a good time to break out the bioweapons, if you have the technology for it. No need to worry about your own troopers catching the disease.

Rick said...

I have no real need to add to this discussion, do I?

Anonymous said...

Rick said:"I have no real need to add to this discussion, do I?"

After you lite the fuse, you're just suppossed to run, aren't you?

If you want to take a planet, why bombard it? Launch an army of robots onto the surface and let them shoot up anything that even remotely looks threatening. Then you deactivate them and land your occupieing force unoppossed.

Of course, I do see a sticking point on all this: the targeted world can use ALL of its armed forces to fight the invaders, but the attacker can only use a portion of his armed forces to invade. Unless he wants to completely strip his planet of all combat units...

I still think that, in the Plausible Midfuture, it will most likely be nation vs. nation during interplanetary wars, and not world vs. world.

Ferrell

Turbo10k said...

"If you want to take a planet, why bombard it? "

Depends on what you want. You want mines and minerals? Drop several ricks of destructiveness (MIRV spam?), preferably on major population centres (the rest are rounded up for forced labour). This would be the case if methane-based aliens are sitting on top of natural deposits of SNCF.
Want the infrastructure (as in the case of same species interaction) ?, Bombard again. This time with Thor bolts and precision laser strikes.

Just to say that 'bombarding' can be quite specific in effect and objective...

Turbo10k said...

Part Three:

On bioweapons:

-Preferred bioweapons will be bacteria on short term invasions, but on major campaigns, viruses or bacteria that hibernate in hard shells will be preferred as have long life and opportunity to mutate better (escaping immune defence reactions and foiling attempts to find cures).

-Just as I noted about bombardment, bioweapons can be quite precise. Current genetic engineering can find highly specialized strains that have the required characteristics; one can destroy the dairy industry and leave the chicken alone for example... More seriously, we could program predetermined deaths into the bacteria's life cycle. Bacteria could be neutralized after the introduction of a second agent into the ecosystem. We could have viruses that are better transmitted through water than air, breaking coastal communities and agriculture while inland habitations wouldn't be affected.

-Finally, the previous have political implications. To force a planet to surrender, I could introduce a debilitating disease that makes the general population generally unhappy. After a few weeks, I propose to distribute a cure for the signature of a treaty. You want said colony to stay heavily reliant on food importations from the homeworld? A nice little virus, engineered from local strains, could be covertly introduced, with nothing pointing towards you.

-Story material: Main character discovers his home is being controlled through benign diseases that affect its economy. Homeworld learns of this and sends undercover agents after him. Main character escapes and accuses Homeworld in front of Colony tribunes. War is declared. Bang Bang Lots of Conflict.

-Lab Wars in SPAAAACE!

Thucydides said...

Having to take over a planet to mine anything (even McGuffinite) seems a tad inefficient. I could probably get more of whatever I want by exploiting asteroids, comets or Kruiper belt objects than going dirtside, and the delta V cost will be a lot lower as well.

Even if the planet is the only source of McGuffinite in the entire known universe [see the movie Avatar for a particularly horrible example..Unobtanium?, Really?]) there are probably more unobtrusive means of obtaining it. In Avatar, a spacefareing civilization that can cross to another star system should be able to tunnel under a tree as the most low tech means (or leach it out using nonomachines or engineered bacteria or just beam it out and replace it with an equal mass of dirt...).

Given the energy and economic costs of long range space travel probably limit many of the traditional causis belli for wars, I would expect that ideology and religion would probably be the only things that would drive large scale conflict in the future. The 30 years war in Europe was particularly destructive, far beyond any rational cost/benefit calculations because it was driven by irrational factors.

Turbo10k said...

Part four:

Aero-spatial fighters:

-Fighters shipped in from space are silly. Three reasons:

1)Will be outmanoeuvred, outperformed and outarmed by planetary fighters since the latter can land to refuel, rearm and don't have to carry all the weighty re-entry modifications (excluding even worse re-exit rockets and fuel).

2) Fighters dropped from orbit will have their position well designated before entry into the aerial theatre. This excludes surprise attacks and stealth, both of which are prime factors in favour of jet fighters today. Air fighters also need either massive amounts of fuel or have very limited flight times. The first is not an option for something shipped in from a million miles away. The second is a major handicap for something that will have to land or go bust. This implies that aerospatial fighter pilots are sent on suicide missions unless they clear a landing spot.

3) Anything a fighter jet can do against land or sea targets is done better by bombardiers in orbit. For the same weight, I can bring several (multiple depends on your setting) cruise missiles that can be dropped into the atmosphere and hunt for opposing fighters inside the atmosphere, without handicaps defined in 1). Why do you need one?

-Nuclear jets are possible and exist. Covair X-6 is an example. While they eliminate the massive logistical problem on jet fuel, or better, could have the capacity to return to orbit, the tons of radiation shielding is bad for performance. Jet heat would also be a problem, as passing air over hot nuclear rods doesn't make for a small IR signature (neither does the air-based cooling system for electric jets. And forget water based cooling systems, I don't see where you could put radiators on a fighter jet)

-Nuclear jets leave huge messes on the ground if they crash or are shot down.

-Depending on setting, surface to air missiles can be an order or magnitude better at shooting down their targets than surface to orbit defences. The latter are supposed to be bad enough o force you out of low orbit for precision strikes.

-I managed to work out a plausible orbital-dropped jet fighter. Mighty expensive though.
It is made out of super-light composite materials with SNCF armouring. Propulsion is done through nuclear-thermal jets (nuclear scramjet) for high altitude and nuclear-electric jets for lower altitudes/speeds. The nuclear-electric jets use electric motors, not resistance-heating fans (like hairdryers) for maximal stealth. It is made to look like an arrow head, but has retractable delta winds and tailplane for lower speeds.
It carries a powerful laser, with a fixed mirror on the back. This means that it has to roll around to aim, and flys up-side-down for ground shots. Adaptive optics compensate turbulence. I chose this position as it maximises mirror size all while avoiding gymballing.
The fighter (more like a laser bomber) is vulnerable to better performing defending fighters; Therefore, it is used in conjunction with high-performance fighter drones equipped with missiles used to keep defending aircraft away.
To enter the atmosphere, it is dropped in along with its drones (15 meter long, around 2-3 of them, look at the BAE TANARIS) along with a disposable heat shield. The shield slows them down and releases them very high so that the combat group can engage in hypersonic flight at very high altitudes. Their strategy would be stay high and fast, with the laser taking down SAMs, attack enemy fighters hiding from orbital bombers from long range, then dropping veeeery low to escape detection when necessary. To exit the atmosphere, the drones are sent off into some suicide mission or assigned to autonomous duties while the bomber drops its wings and turns into a rocket.

How do you like it?

Turbo10k said...

"Given the energy and economic costs of long range space travel probably limit many of the traditional causis belli for wars, I would expect that ideology and religion would probably be the only things that would drive large scale conflict in the future. The 30 years war in Europe was particularly destructive, far beyond any rational cost/benefit calculations because it was driven by irrational factors."

Oi! Who said war was rational in the first place?

Cheery Reaper said...

Indeed. There will always be powerful and greedy people who will start a war for anything.

Atomic Rockets has an entry that suggests asteroid mining may not be that practical, and with terraforming and lengthy ardous process people will still want to conquer habitable planets.

Raymond said...

Turbo10k:

Nuke-thermal jet engines don't need a separate radiator - the airflow through the turbines is the radiator, and the heat cycle will be very similar to a jet engine (Brayton cycle, similar thermal range). In fact, the turbines used would be basically conventional turbines with a heat exchanger instead of a combustion chamber. The throttle curve will be very different, though, due to the relatively fixed heat output.

I also don't see nuke thermal being as useful for fighters (for many of the reasons pointed out) as they would be for air transport for ground troops (and without the fuel concerns, very useful indeed). Or hovertanks - not being tied to fuel may make them actually plausible for once. Radioactive debris isn't quite as much of a concern as you'd think; NERVA tests in the 60s found solid-core engines would crash relatively intact as opposed to turning into a cloud of radioactive dust.

The performance advantages of planetary-only fighters, however, will be limited by the threat of orbital lasers (at least on clear days or above the clouds). Weather still matters.

Milo said...

Also consider the merits of nuclear-powered aircraft for the defender. Airports are rather vulnerable to orbital bombardment so you'd like to be able to keep your planes up and moving.

Nuclear-powered aircraft would be a lot easier if you have fusion rather than fission.



Ferrell:

"If you want to take a planet, why bombard it? Launch an army of robots onto the surface and let them shoot up anything that even remotely looks threatening."

The enemy has more and better-armored robots than you.

Also, that sounds like a rather roundabout way to cause indiscriminate destruction.

Milo said...

Turbo10k:

"[...] will be preferred as have long life and opportunity to mutate better"

"More seriously, we could program predetermined deaths into the bacteria's life cycle."

But you want the bacteria to mutate and evolve. The ones that mutate to not die when preprogrammed will fare better than the ones that didn't mutate to not die, so those will end up taking over the population.


"Bacteria could be neutralized after the introduction of a second agent into the ecosystem."

Ooh. Maybe we could design a bacterial bioweapon to target the bacteria?


"We could have viruses that are better transmitted through water than air, breaking coastal communities and agriculture while inland habitations wouldn't be affected."

Because, of course, desert-dwellers are well known for never drinking water, taking baths, or washing their hands.


"After a few weeks, I propose to distribute a cure for the signature of a treaty."

And then the cure turns out to not work because the bacteria mutated to foil it. You wanted them to do that, remember?

Cheery Reaper said...

On Mecha.

While I think thirty meter tall mechs are as bad an idea as the WWII Maus-

A short Mech might be of use.

I'm thinking an Apache Gunship with legs and two grips for mangling people, and it wouldn't be any taller than a WWII M-4 Sherman, an eight-nine foot mecha is still awesome.

However, even if you solved the high profile issue there's the problem of utility. If you have tank which can mount more weapons and more armor and a shorter profile, what's the point? Make a mech any smaller and you end up wearing it thus getting power armor.

Anonymous said...

Replying to the obsessed Scholar: on Mecha...there exists now a walking machine about 8' tall and 20-30' long; it has six legs and grippers. It was designed for low-impact logging in Oregon, but I'm sure someone could modify it for war.

Oh, and just as an aside; half the things I say here are to spur the other readers to comment.

Ferrell

Thucydides said...

SUSTAIN was a USMC proposal from 2002 to launch Infantry squads on suborbital trajectories to insert anywhere on Earth in under 2 hours. Think of SpaceShipOne landing and disgorging a squad of Marines for a mental picture.

The troops would probably blow the ship on the ground and be extracted by other means after the mission.

If the ships and troops were housed on a space station for global coverage 24/7, then a disposable areoshield (maybe even an inflatable one) would be needed for re-entry, but the ship could then glide a long way for insertion after ejecting the shield (a small jet engine might also be added to improve performance of the ship). The space station itself would be pretty vulnerable, so it would have to be capable of orbital manouevres, and possibly capable of traveling through cis-lunar space for strategic depth and the ability to evade ASAT weapons. Now it is a spacecraft.

Voila: a Space Navy and Marines

Turbo10k said...

"Nuke-thermal jet engines don't need a separate radiator - the airflow through the turbines is the radiator"

I know that...I was talking about the nuclear electric ones...

"But you want the bacteria to mutate and evolve."

Only if I want to wipe out the whole species. If I don't, I'll make a bacteria that is set to not mutate, AND have a preprogrammed life cycle. I think eliminating mutations requires simpler bacteria, ones that have low reproductive rates (cuts into infection spread" all while having DNA/RNA that better repairs itself.

"Because, of course, desert-dwellers are well known for never drinking water, taking baths, or washing their hands"
All the worse as their limited water supplies are taken over quicker.

Working backwards, we could have a bacterial time bomb, which is inoffensive at first, but becomes mortal or died depending on the second agent you introduce.

On Mecha: Why the fixed, bipedal image of mecha? One design I like is a six-legged, low profile thing with wheels at the end of its legs. This gives it the best of both worlds: offroad capability, low profile, heavy armor AND fast onroad speeds.

And finally;

Aerobraking ships:

-Useful for faster intercepts. You approach at higher speeds, reach the bombing zone lower and faster, then escape on a high elliptic orbit.

-Lower drop pod releases mean they spend less time in the air, so are less likely to be shot down.

-A good way to release something without having a huge, fragile heatshield. I'm thinking aerospatial fighters here, as they can enter hypersonic regimes immediately.

-Closer ground support? I'm suggesting lasers which would have less air to penetrate...

Thucydides said...

You are proposing lot of mass and equipment for an aerospace fighter.

The Common Aero Vehicle proposal is for a very small craft (maybe 4000 lbs) that can be launched by a repurposed ICBM booster. After re-entry, it manouevres and ejects its weapons out the back, delivering smart bombs or similar ordinance to the target.

Hypersonic waveriders confine the shockwave and plasma under the vehicle, and could theoretically carry weapons or cargo on the upper surface, or eject them from the back as well. Oddly, a waverider could carry a laser with the mirror on the top deck to attack re-entering ships or objects in LEO, so this might be the form of the defender's aircraft!

Turbo10k said...

"You are proposing lot of mass and equipment for an aerospace fighter."

With a single fighter being able to cover the globe very quickly and with infinite endurance (okay okay, but on the timescale of a single campaign, it is not a lmimiting factor), the number of craft needed is greatly reduced as increasing numbers does not contribute to increasing presence or coverage on the field. A few hypersonic nuclear jets are all that is needed. At 35 tons each, less if they have exclusively laser weaponry, they won't make up a great fraction of the attacker's surface forces...

Lasers mirror on the back:
Pausible even for a ground-attacking waverider. The craft can slow down, flip around and drop as it loses lift, fire, then return to cruising position. Complicated, I know, but solves the problem of a mirror being on the underside...

Milo said...

Another limitation on an aircraft's range is the human factor. Airplanes have even less roomy accomodations than submarines.

Thucydides said...

A hypersonic waverider packing a laser (or any laser platform for that matter) simply needs access to cooperative relay mirrors to attack targets outside of the primary mirror's arcs.

The relay mirrors can be on any platform, including other waveriders, UAV's, blimps or satellites in LEO, vastly complicating the defender's calculations, since it will be very difficult to locate the primary emitter or protect against shots which are coming in at unpredictable angles. Swarms of relay mirrors are individually low cost, low value targets, so shooting down the mirrors is also a difficult proposition at best (and while you are busy doing that, the primary emitter could be coming over the horizon to get you...)

A high tech laser slugfest could be a bit like being in a disco...

Raymond said...

Thucydides:

I'm going to invoke Rick's Second Law: if laser mirrors are cheap, you aren't building them big enough.

Milo said...

Speaking of mirrors:
- How well will a laser work while it has some holes in its mirror? In particular, how will this affect focussing?
- Just where do you put the mirror anyway? Mirrors reflect, so I'd think the laser emitter and your target would have to be on the same side of the mirror.

Luke said...

Milo:

A hole in your mirror will reduce the light reflected by the reduced area available for reflection, and by diffraction around the hole. In the far field, these losses are equal. Small holes or scratches will have a small effect on a large mirror, while if a significant fraction of the mirror's surface is degraded you will see serious reductions in beam quality.

The mirrored beam pointers are usually set up similar to modern reflector telescopes. These include a stationary laser, a series of mirrors to reflect the beam to a secondary mirror that expands the beam to be reflected off the primary mirror. the primary sends the beam on its way to the target. Winch Chung goes into some detail on Atomic Rockets http://www.projectrho.com/rocket/rocket3x1.html#turret

Anonymous said...

However you design your aerospace fight, I'd rather have them than not; they give you flexability and a way to directly counter the enemy fighters (whether aerospace or just air-breathing); If you do have to land troops, you want them to have air cover.

Ferrell

Thucydides said...

All other things being equal; an attacker coming in from space will be better off concentrating his available mass to orbital platforms capable of attacking ground targets, rather than diluting the effort by bringing aerospace fighters.

For 35 tons, I get a honking big mirror and Liniac or Surfatron to drive a FEL weapon, while a 35 ton nuclear Aerospace fighter has a much smaller mirror and liniac/surfatron (very much smaller; in the here and now it takes a 100-200 ton cargo plane to carry a laser of militarily useful size.

Any defender would want something like this sort of aircraft to prosecute targets, and if it is the carrier vehicle for a relay mirror, then the emitter could be a weapon of enormous power.

Milo said...

The fighter has the advantages of operating under clouds, and of having much better freedom to move around rather than following fixed orbital tracks.

Turbo10k said...

"The fighter has the advantages of operating under clouds,"

The main reason we are evoking aerospatial fighters, as laser ships from orbit can see so can't hit craft hiding under cloud.

"and of having much better freedom to move around rather than following fixed orbital tracks."

I'd give anything for a craft which is in the slightest unpredictable. What's the point if I can get a 35 ton superlaser in space if a single small kinetic shatters it? Also, if I can have a 35 ton superlaser in space, how comes the minimum mass of a superlaser-equipped waverider in 200 tons?! Not like we'd need a superlaser in the atmosphere. We can content with much smaller mirrors and less power output since:
-We're closer to target.
-We only have to go through a single medium. That means a laser spaceship which needs seperate near-IR and green lasers for battle can be split into aircraft carrying green lasers while the spacecraft only carries space combat lasers.
-Cooling systems would be considerably smaller, and easier to maintain (lower operating temperatuires, cheaper materials....

At worst, we could have waveriders carrying big mirrors on their backs, redirecting laser beams to target. The laser bomber could stay in a higher orbit, safer, while its beams are more precise and more damaging due to the relay. A plane is also harder to shoot down than a reflective balloon (again, depends on cost ratios).

Turbo10k said...

On kinetic penetrators:

The design which has been condensed from the last 600 posts is a tungsten rod 3 cm wide and a meter long, with a carbon head that has to strike a balance between slowing the rod down and penetrating better. More advanced designs could incorporate a detachable head to deal with Whittle-shield like armor.

I propose the perfect solution: A blunt, heatshield efficient carbon head that...splits in two like a discarding sabot to leave the sharp tungsten penetrator behind it with the required, pointy shape...

Turbo10k said...

On PA:

Electromuscles deal with lots of the size and power/weight ratios of the larger electric motor/pulley systems. Please forget hydraulics because even if the have nice lifting capabilities for their size, they are too slow and complicated, not something you want in a battlefield...

Albert said...

Turbo10k said...
if any nation from 1 to 199 decides to help you, you have your well-established gap in the global defences without any bombardment required. Anti-orbit weaponry can still cover other nations, as happens today. The same applies (although to a far lesser extent) for anti-air defences.
Also, you are assuming that the remaining 198 nations don't decide to collectively invade and spank the nation that decided to help you before you get in orbit.

Just to say that 'bombarding' can be quite specific in effect and objective...
Sir, I think you fail. Neutron bomb carpet bombing will safely sterilize population centers, and you can dump lots of small-sized bombs (tactical nukes) to overcome defenses.

Oi! Who said war was rational in the first place? In most occasions in the past (up to WWI), war had been waged to plunder the wealth and the territory of the enemy.
In those situations, war was an investment. Today is primarily a loss of money due to ludicrously overpowered weaponry.

I know that...I was talking about the nuclear electric ones... that add to the craft all the coolant's weight, the turbines the alternators and whatever. And RTGs are a tad inefficient to produce enough power for applications like turbofans.

Milo said...
Incidentally, war with aliens is a good time to break out the bioweapons, if you have the technology for it. This assumes that you manage to throughly understand alien's biology at an unreasonably fast rate. 10 years would be the most bare minimum time to understand the basics of a not-so-different-species (one that uses ammonia instead of water), while for more alien stuff that has a totally different non-protein-based biochemistry (imho more likely), you're lucky to get away with 100 years.

If aliens have bumpy foreheads and have the same progenitor (ala Star Trek), then yes, bioweapons become a major threat.

Also consider the merits of nuclear-powered aircraft for the defender. Airports are rather vulnerable to orbital bombardment so you'd like to be able to keep your planes up and moving.
VTOL aircraft can solve that.
And the main problem I forsee for them is the obvious neutrino signature if they have full shielding (wishful assumption) or add a strong ionizing radiation source that you cannot hide when in flight if you just have a shadow shield (more realistic imho).
Fusion reactors help with the shielding by choosing the appropriate fuel, but neutrino emissions will still be fairly detectable.

I know that neutrino sensors are pretty big and inefficient today (although can already locate nuclear power plants), but if you can have a viable nuclear reactor (or even a fusion one) on your aircraft it's reasonable to assume that neutrino detectors will improve as well.

-Albert

Albert said...

Thucydides said...
In Avatar, a spacefareing civilization that can cross to another star system should be able to tunnel under a tree as the most low tech means
In the original Avatar concept (Project 880, a good read, google it), unobtanium was in the flying mountains, and they wanted to drive off the oversized smurfs that were protecting the mountains because they were holy to them (tree of souls).
Then the above was dumbed down in Avatar to allow even 2 year olds to understand it at first glance.

Given the energy and economic costs of long range space travel ... I would expect that ideology and religion would probably be the only things that would drive large scale conflict in the future.
And just as with Hitler, you risk to incur in the fear of the rest of the nations that then unite to stomp you.
When you go to war you generally don't want to sound threatening to to other guys.
But ideology and religion as you said are convenient escapes from this cold rational logic.
It's just that unless you are the most powerful around, you'll get stomped by Galactic Alliance sooner or later.

Raymond said...
I'm going to invoke Rick's Second Law: if laser mirrors are cheap, you aren't building them big enough. There is also the point of all the air that lasers must travel through. You need to account for much bigger energy losses. And for stuff getting in the way, like clouds, fog, dust.

-Albert

Albert said...

On Space-dropped aircraft:
ignoring technical details like attacker's payload mass considerations and endurance of the craft, I think attack aircraft have only one real role (as most today's fighers after all) bombing stuff on the ground when your orbital guns cannot do it.

This means shooting from below clouds mostly, because if the defender has anti-orbit defences that can threaten laserstars I don't see how reentring aircraft would be able to get through in one piece.

Fighting other fighters isn't exactly a primary consideration, because missiles already reduced the Romancs's heroic dogfights to something like "point and shoot", and creative evasive maneuvers to "release chaff" and "outrun the missile". Lasers won't help this either.
Anti-air missiles or anti-air lascannons are dirt cheap (compared to a fighter mounting more or less the same weapon) so you can fill your planet of them and use them as your primary anti-fighter weapon.
If really annoying, fighters can be taken down by launching airburst tactical nukes, and the decently large AoE shockwave kills the aircraft.
(please remeber, if you have a fighter is because your laserstars cannot shoot there, so they cannot cover the fighter's sorry backside too)

For the defender instead, aircraft are pretty useful. They can hide behind clouds, form clouds where needed or keep them from disappearing, then "surface" from clouds, shoot lasers and get down again.
Other than that, aircraft are useful to redeploy stuff fast, thus exposing it for less time to risk of laserstar fire. No, I don't think you will be able to keep a constant eye on every friggin square cm of the surface (not to mention they can hide under clouds too).

I don't favor nuclear aircraft for the defender. Nuclear stuff is tricky to mantain, heavy (radiation shielding, fissiles) and *much more* expensive than a comparable engine that uses more common fuels.

Greater numbers and easyness of service will be the defender's major strenght, while the attacker hasn't much choice and will be forced to take what gives him most bang for the mass.

Also, placing underground/undersea refineries close to the extraction point of the fuel (underground/sea too), thus protected from orbital eyes/bombardment isn't that much harder than making a working nuke-powered aircraft.

-Albert

Geoffrey S H said...

There may be no gap in the defences, but there would be less stuff to take out as opposed to a planet occupied (and defended) from the same power.

Also, there is less need to take out stuff directly below/ in front of you, somewhat helping your chances.

Not much of a difference Albert, but I hope its a helpful difference nonetheless.

as concerns fighters, small pds lasers might prove useful against missiles (VERY small), with some aircraft being glorified counterbattery lasers operating under cloud cover.

Bad example, but something to chew over.

I'll probably be told it won't work, but semi-submersible ground-effect ships- might they have some use? Fast to try and dodge kinetics, but able to dive under the surface if there is a laser incoming. No, its not foolproof nor entirely workable, but still something to chew over.

p.s.: They'd carry a laser of missile to use against spacecraft.

Geoffrey S H said...

*ed: ps that should be laser OR missile (I.e: smal ICBM.

One post I put which was eaten talked of mobile space-elvators to get equipment to the ground- try to shoot it down once it has impacted and it wraps round your planet, not a good idea. Yes, I know it would not work, but it was in a reply by the obbsessed scholar as to this problem, and again, it might provide some replies that come up with better solutions or niche ideas tenuously related to this concept.

Milo said...

Turbo10k:

"I'd give anything for a craft which is in the slightest unpredictable."

That's useful too, although I was thinking more in terms of responding to enemy movements. If you're overly predictable, the enemy can deliberately go to where they know you won't have fire cover anytime in the next 12 hours.



Albert:

"This assumes that you manage to throughly understand alien's biology at an unreasonably fast rate."

I did say "if you have the technology for it".

I also didn't say that you necessarily went to war with the aliens the moment you met them. You could have been having peaceful relations with said aliens' culture for quite some time before hostilities began. That would give you time to learn their biology (and vice versa).

Chances are that any war we have with aliens will not be about "OMG they're weird and different, destroy destroy destroy!" - on either side.


"It's just that unless you are the most powerful around, you'll get stomped by Galactic Alliance sooner or later."

One or the other side getting stomped is a pretty common conclusion to wars. The question is how good a story you can tell about what happens before the shoe comes down.


"ignoring technical details like attacker's payload mass considerations and endurance of the craft, I think attack aircraft have only one real role (as most today's fighers after all) bombing stuff on the ground when your orbital guns cannot do it."

And shooting other planes (which are still "ground targets" from the point of view of a space force), and reconnaissance (which is not technically an attack role). Which are basically all the roles orbit-to-surface spacecraft will be performing in the first place.


"Fighting other fighters isn't exactly a primary consideration, because missiles already reduced the Romancs's heroic dogfights to something like "point and shoot", and creative evasive maneuvers to "release chaff" and "outrun the missile". Lasers won't help this either."

Just that it's boring doesn't mean it won't happen, although we might not dwell on the details much. But yes, lasers make for some pretty boring dogfights. Laser fighter tactics will involve hide and ambush - lurk behind clouds, behind mountains, if all else fails then low on the ground and behind the planet's curvature - and try to find the enemy before the enemy finds you. Like sniper duels.

Most air-to-air combat, though, would likely involve the attacker's hunter-killers seeking out the defender's surface-to-orbit planes. This would not be combat between equal fighters. (Even if they can point their laser horizontally, surface-to-orbit planes will probably take after our interceptors somewhat, being designed more for speed than agility.)

Thucydides said...

If your airforce is confined to hiding below clouds, then I expect the attackers have some freedom of movement (and can drop pods of AAM's into cloud banks if there is a real threat).

A 35 ton laser battery is probably an SFnal "laser of stupendous range" and can identify and prosecute targets on the ground from well beyond GEO if needed. This would be part of the constellation of attacking spacecraft, so it has already served a useful purpose in vapourizing the enemy space fleet and defenses. If it is an FEL, then the wavelength can be tuned to whatever works best in the local atmosphere, and issues like hiding behind mountains become moot.

Albert said...

Geoffrey S H said...
There may be no gap in the defences, but there would be less stuff to take out as opposed to a planet occupied (and defended) from the same power.
Sure, you can craft this situation carefully: the nation helping you must have a good phisical size, (China is ok, Switzerland isn't) it must be able to fend off ground assaults from other nations by itself, or able to keep the secret till the last moment.

Otherwise it gets militarily invaded and filled with "trucks of missiles" that are pretty much all you need for a decent defence.
Won't probably have any laser cannons left to snipe at you (they would have self-destroyed them during the attack), so that area may still be a weak point, (relatively speaking, probalby it will have much heavier presence of ground troops to repel the probable ground troop drop)

Not much of a difference Albert, but I hope its a helpful difference nonetheless.
This can make a huge difference as cannot make any. All depends from who is your enemy and who is your ally. Author's choice, as always.

as concerns fighters, small pds lasers might prove useful against missiles (VERY small)
Missiles are unbelievably flimsy.
You can kill about any missile with an attack rifle's level of firepower.
It's the "shooting on something ludicrously fast at a huge range" that cannot be done by a modern attack rifle.
A laser rifle of comparable firepower with a bigger mirror (better range) and good automated targeting system (better aim) can kill pretty much any missile you care to throw at it.

This will start a game of missile-armoring or cheap missile spam to compensate, and if the laser is good enough (technological assumptions) they may disappear completely.

I'll probably be told it won't work, but semi-submersible ground-effect ships- might they have some use? Water is an awesome armor against supersonic weapons. You just need a sub that can dive *fast*.
The craziest idea would be a submersible cavitation hypersonic VTOL fusion-powered aircraft.
Now THAT is what I wanna see in mah future.

-Albert

Albert said...

Milo said...
I did say "if you have the technology for it".
I was claryfing that statement by giving time figures to aquire the necessary knowledge for it.

Chances are that any war we have with aliens will not be about "OMG they're weird and different, destroy destroy destroy!" - on either side. Human history proves otherwise, and there is some interesting discussion of this in the Atomic rocket's page about aliens.
Here

One or the other side getting stomped is a pretty common conclusion to wars.
If the leader is a moron, yes.
But you can also make him act smart and try to not piss of other space nations to avoid getting stomped so easily (the difference between Hitler and Fancisco Franco).
Just to not make the story look like WWII too much.

and reconnaissance (which is not technically an attack role)
That is more a drone's work, but I don't think a drone will be as useful as today against a well-armed defender.
High altitude drones will have to use clouds as cover or risk being shot down (so will only make a few sneak peeks and then get into cover again), low altitude ones can be ECM'd with ease.

This would not be combat between equal fighters.
Bombers don't usually dogfight.
There will be more than one type of aircraft, a "reverse bomber" (that hypersonic guy with the laser mirror on top), whatever you mean for surface-to-orbit fighters (imho they are a silly vehicle) and a more conventional fighter meant to escort the bomber to engage the enemy's aircrafts.
And the ground defences of course.

Thucydides said...
A 35 ton laser battery is probably an SFnal "laser of stupendous range" Range depends from mirror's size. I doubt you can get a mirror bigger than 10 meters AND a decently powerful laser engine with that weight constraints.

-Albert

Milo said...

Albert:

"Human history proves otherwise,"

No, it doesn't. To take the usual example: the European settlers in the Americas were motivated by a desire for land and gold, not by an ideological mandate for genocide. The genocide was just incidental, to get rid of people that got in the way of their economic interests.

We might go to war with aliens immediately upon meeting them, if they happen to have something we want. Although it's not clear why that would happen, since we won't want the land of ammonia-breathers and their planets won't have more gold than our asteroids. We definitely won't be interested in their women.

But most wars are started by rising political tensions between factions that have already known about each other for some time, or at least that are familiar with each other's culture.

Thucydides said...

While the engineering of SFnal equipment is beyond my pay grade, high energy FEL's will probably use "Surfatrons" (Plasma Wakefield Accelerators), which can accelerate electrons to huge energy levels in millimeters rather than the meters to kilometers of conventional LINIACs.

Mirror size in space can be fairly arbitrary, while a 10m mirror sounds like the standard, there is no reason larger mirrors can't be made so long as they can maintain their optical properties. Other means of focusing beams exist (Robert Forward used to write about gigantic Fresnel lenses to focus lasers on lightsail starships), so the limit in space is more a matter of logistics than anything else.

Even if we grant that aircraft, ships and ground units can be protected by cloud formations, that still puts a huge crimp in the defender's ability to take action; rather like the Vampires in Twilight can only live in a very restricted geographic range where the sun is blocked most of the time. (yes, I have a daughter who is into that...)

Luke said...

Turbo10k:

The design which has been condensed from the last 600 posts is a tungsten rod 3 cm wide and a meter long, with a carbon head that has to strike a balance between slowing the rod down and penetrating better.

Well, that's one of the two designs. The other is a 3+ meter long tungsten rod that barely slows down and slams into the ground with enough explosive force to take out fixed instalations.

I propose the perfect solution: A blunt, heatshield efficient carbon head that...splits in two like a discarding sabot to leave the sharp tungsten penetrator behind it with the required, pointy shape...

That was kind of the idea I had in mind. Have a wide, blunt cone that slowed the penetrator down to about 2 km/s to 2.5 km/s, and then falls off.

Sharpness isn't really an issue, so long as you maintain the length of the rod. At these speeds, the tip will flow like a liquid no matter what its original shape, so a pointy end doesn't really help.

Luke said...

Albert:

Fusion reactors help with the shielding by choosing the appropriate fuel, but neutrino emissions will still be fairly detectable.

If you have aneutronic fusion reactors, you will not be generating any neutrinos. Even if your reactor produces neutrons, the reaction itself proceeds via the strong nuclear force so no neutrinos will be emitted by the fusion process. Neutrinos will be emitted by the beta decay of neutron activated materials (and will continue long after the reactor is shut off), so you can limit the neutrino emissions by using materials that are not easily activated.

I know that neutrino sensors are pretty big and inefficient today (although can already locate nuclear power plants), but if you can have a viable nuclear reactor (or even a fusion one) on your aircraft it's reasonable to assume that neutrino detectors will improve as well.

Fusion is a matter of plasma physics. There's no fundamental reason to think compact fusion reactions are impossible (although I tend to be skeptical about fusion power at all in the plausible mid-future).

Neutrino detection, on the other hand, is a matter of the physics of fundamental particles. To get a significantly more compact neutrino detector, you would need to increase the fundamental coupling of the weak nuclear force. This is going into the realm of superscience.

Luke said...

Geoffrey S H:

One post I put which was eaten talked of mobile space-elvators to get equipment to the ground- try to shoot it down once it has impacted and it wraps round your planet, not a good idea.

Space elevators made out of "real" materials (like super carbon nano-stuff)would not wrap around the planet if broken. They would be under enormous tension, and if severed the tension release wave would disintegrate the nanotube cable into carbon fluff.

Luke said...

Thucydides:

If it is an FEL, then the wavelength can be tuned to whatever works best in the local atmosphere

Note that while FELs are tunable over a limited range of wavelengths, you would need a separate beam path and wiggler to get a near infrared or visible beam out of an FEL designed for x-ray laser wavelengths, not to mention a different set of focusing devices. You still get to use the same linac, so you do get some cost and mass savings.

Luke said...

Thucydides:

high energy FEL's will probably use "Surfatrons" (Plasma Wakefield Accelerators), which can accelerate electrons to huge energy levels in millimeters rather than the meters to kilometers of conventional LINIACs.

Wakefield accelerators have their use - but they are inherently low efficiency, which rather limits their applications for weapons, where getting a high power beam out is of paramount importance. A linac using superconductive cavities can be quite efficient. However, there is still a lot of room for improvement in conventional linacs. Decreasing their size by at least an order of magnitude would be quite plausible in the near future. I would even accept two orders of magnitude with the right handwaving.

Albert said...

Milo said...
We might go to war with aliens immediately upon meeting them, if they happen to have something we want.
*cough* Avatar *cough*
There is also the risk that aliens will attack first for unknown reasons.
And that only if you attack first you will manage to survive.
That will surely mean instant cold-war situation until you manage to know them better. (and knowing better their psyche and emotions, not just their tech)
And that by itself can become a good reason for war if the aliens act strange and seemingly threatening while in fact they were just snooping around to know you better.

In the old days talking with URSS was possible, US had spies within URSS (and the reverse), and they were humans so most reactions were more or less predictable.
Any true alien race won't give you these benefits. There will be fear and paranoia, lots of it. From both sides.

Thucydides said...
Even if we grant that aircraft, ships and ground units can be protected by cloud formations, that still puts a huge crimp in the defender's ability to take action;
Well, weather control is still in its infancy (we can seed rain or keep rain from falling), so with some "future advancement" they may create clouds where needed when needed.

Otherwise they must rely on huge smoke generators to nullify lasers.
Their movements will be obvious, but the attacker won't be able to do a lot about it (if kinetics can be dodged, of course).

Or a planetwide tunnel network, or good antisensor lasers that fry enemy's camera optics.

But yeah, if the attacker just wants a genocide, all the above are pretty moot. Those tricks can hide your military, but not your civilians (unless quite abused).
That situation simply comes down to
"Does the attacker overcome active defences?"
YES= nearly genocide, only a few secret underground vaults survive.
NO=gets back home

In case the enemy wants to capture the planet with the population intact (let's say Rebel Alliance that is trying to free a Evil Empire's planet, to keep the reasons ideologic), then you can make the war last a little more by using the above tricks.

Luke said...
a 3+ meter long tungsten rod that barely slows down and slams into the ground with enough explosive force to take out fixed instalations.
What I always imagined for orbital bombardment. Isn't that the Thor?

To get a significantly more compact neutrino detector, you would need to increase the fundamental coupling of the weak nuclear force.
I was thinking that by the time you can place a fusion reactor on an aicraft (quite a feat, considering the slow pace of fusion tech development) someone may have found a workaround, like that article about nuclear decay influenced by the distance from the sun.
In other words, aircraft with onboard fusion reactors are very very close to the Era Of Magitech to me.

About space elevators:
That's interesting. All I read was about the cable either escaping into space or burning in reentry or even graciously falling down due to low density (depending from where it is severed).
Not that they were overly reliable sources anyway (mostly wikipedia and novels).

-Albert

Albert said...

Milo said...
We might go to war with aliens immediately upon meeting them, if they happen to have something we want.
*cough* Avatar *cough*
There is also the risk that aliens will attack first for unknown reasons.
And that only if you attack first you will manage to survive.
That will surely mean instant cold-war situation until you manage to know them better. (and knowing better their psyche and emotions, not just their tech)
And that by itself can become a good reason for war if the aliens act strange and seemingly threatening while in fact they were just snooping around to know you better.

In the old days talking with URSS was possible, US had spies within URSS (and the reverse), and they were humans so most reactions were more or less predictable.
Any true alien race won't give you these benefits. There will be fear and paranoia, lots of it. From both sides.

Thucydides said...
Even if we grant that aircraft, ships and ground units can be protected by cloud formations, that still puts a huge crimp in the defender's ability to take action;
Well, weather control is still in its infancy (we can seed rain or keep rain from falling), so with some "future advancement" they may create clouds where needed when needed.

Otherwise they must rely on huge smoke generators to nullify lasers.
Their movements will be obvious, but the attacker won't be able to do a lot about it (if kinetics can be dodged, of course).

Or a planetwide tunnel network, or good antisensor lasers that fry enemy's camera optics.

But yeah, if the attacker just wants a genocide, all the above are pretty moot. Those tricks can hide your military, but not your civilians (unless quite abused).
That situation simply comes down to
"Does the attacker overcome active defences?"
YES= nearly genocide, only a few secret underground vaults survive.
NO=gets back home

In case the enemy wants to capture the planet with the population intact (let's say Rebel Alliance that is trying to free a Evil Empire's planet, to keep the reasons ideologic), then you can make the war last a little more by using the above tricks.

Luke said...
a 3+ meter long tungsten rod that barely slows down and slams into the ground with enough explosive force to take out fixed instalations.
What I always imagined for orbital bombardment. Isn't that the Thor?

To get a significantly more compact neutrino detector, you would need to increase the fundamental coupling of the weak nuclear force.
I was thinking that by the time you can place a fusion reactor on an aicraft (quite a feat, considering the slow pace of fusion tech development) someone may have found a workaround, like that article about nuclear decay influenced by the distance from the sun.
In other words, aircraft with onboard fusion reactors are very very close to the Era Of Magitech to me.

About space elevators:
That's interesting. All I read was about the cable either escaping into space or burning in reentry or even graciously falling down due to low density (depending from where it is severed).
Not that they were overly reliable sources anyway (mostly wikipedia and novels).

-Albert

Thucydides said...

I notice that the vast majority of the posts have been more about fire support rather than ground combat per se.

This plays into one of the oldest fails in military history, the wonder weapon that will allow your side to roll over the enemy and prevail with minimal casualties and cost to you. War elephants, catapults, chariots with scythes on the wheels, fire arms, rifled fire-arms, rapid fire cannon, machine guns, gas, aircraft, tanks, nuclear weapons, cyber attack...the list is full of every practical and impractical idea that people come up with.

In the end, it is a combination of clear objectives, willpower (military and political), logistics and the tactical leadership of the troops which bring victory.

Churchill would not quit, and knew the resources of the Empire and the Americans were available to be tapped, he knew without a doubt that the British Empire would emerge victorious in three wars (WWII can be thought of as a contest between the Western Allies and National Socialist Germany in the European theater, the Empire and Fascist Italy in the Mediterranean sea and a clash of Empires in the Pacific and Indian Oceans, as well as a titanic land struggle between the National Socialists and the Bolsheviks in Eastern Europe and the Japanese Empire and China in the East.)

In the end, it didn't matter in the long run how great German tanks were, the British could build or buy far more, and use their control of the seas to bring tanks and other war materials wherever they were needed. The United States had the same advantages plus a social system which encouraged more innovation and eventually the ability to outdo even the Empire.

As for wonderweapons: http://www.youtube.com/watch?v=zzoeEdW-EDQ

Milo said...

Albert:

"There is also the risk that aliens will attack first for unknown reasons.
And that only if you attack first you will manage to survive."


I am not in the habit of breaking into random people's homes and assaulting their stoves with buckets of water just in case they might cause a dangerous fire, and neither should you.


"And that by itself can become a good reason for war if the aliens act strange and seemingly threatening while in fact they were just snooping around to know you better."

I agree that "mutual misunderstanding" is the most likely cause of war between newly met species. Even accounting for convergent cultural evolution, aliens will have very different social norms from us, and so they might unknowingly do something that we consider threatening - or vice versa. Of course, both sides will probably be aware what they're getting into and will know that they should expect weird but probably-well-meaning behavior from the other party, so I expect they'll try to be patient rather than drawing guns at the first faux pas - but even so, our patience is finite. We can't guarantee we'll manage to avoid escalating our inevitable early mistakes.

Ender's Game is a popular example, what with the hive mind that didn't realize an individual human life would hold any value - and when it later did realize its mistake, didn't know how to attempt reconcilliation talks.


"Any true alien race won't give you these benefits. There will be fear and paranoia, lots of it. From both sides."

Also curiosity. Many of us have spent so long desparately searching for someone to talk to, that we're highly unlikely to just blow them up immediately when we've finally found them.

And however alien they are, curiosity is an emotion any intelligent tool-using species is likely to share. If they didn't care to study stuff, how did they develop the technology for interstellar travel in the first place?

So that gives us at least one common ground with whoever we end up meeting. That gives us something to work with.

Milo said...

Albert:

"Their movements will be obvious, but the attacker won't be able to do a lot about it (if kinetics can be dodged, of course)."

Airplanes are fast and agile enough to be pretty much immune to any unguided kinetics (from orbital altitude) they can see coming. Since a kinetic you can't see coming is pretty hard to do, that leaves lasers or guided missiles.

Ships are slower and larger, and can get hit with a shockwave from a near miss hitting the water, so are somewhat vulnerable to unguided kinetics, and submarines can avoid any kinetic that isn't also a torpedo by submerging. Bunkers are more vulnerable to kinetics - you need a pretty big boom to bust them, of course, but a big boom can be managed, while zapping bunkers with lasers isn't going to accomplish much.


"I was thinking that by the time you can place a fusion reactor on an aicraft"

If you have fusion reactors on your spacecraft, then it's not that much of a stretch to fit one on an aircraft as well.

If you don't have fusion-powered spacecraft, of course, then yeah, you're not getting fusion-powered aircraft either.



Thucydides:

"This plays into one of the oldest fails in military history, the wonder weapon that will allow your side to roll over the enemy and prevail with minimal casualties and cost to you."

The problem with these wonder weapons is that when they really are anywhere near as good as advertised, your enemy will have them too.

It's lot of fun imagining how a space armada can send puny land forces scurrying away from their terrible bombardment from unassailable heights, but the truth is that this only works when the land forces are from a non-space-age civilization.

Geoffrey S H said...

Ah, I trust Kim Stanely Robinson (Red Mars) too much. :( He had a cable wrapping round Mars when the elevator got shot down.

For bits lower n the athmosphere though, might you want bits falling down if it disintigrated. Perhaps some anti-ordinance defnces on the cable might be needed. Drat, up goes my payload limit.

Albert- I like the aircraft concept, though I went for a ship because of the inherant stability with such a design. I like the idea of a weather control system, but the speed of the thing might allow some moderate cloud-"hopping" strategies, with it diving underwater when there are no clouds.

Much of my setting depends on counterbattery lasers being very efficient- better to take small pds lasers (that areeasily replaced when eyeball fried) and large "tough" kinetics than large (hard to replace when eyeball fried) offensive lasers and small "weak" support kinetics. Thus, some form of mobility (supported by counter-battery laser planes) is possible for ships and aircraft (and air-ships!!). Like you said Albert, it depnds on the author. Speaking of settings, pos might be intersting to read the settings of various posters in more detail one day...

Cheery Reaper said...

There's one problem with the idea of winning via SLAG, Orbital Strikes, etc.

Enemy: "Mwahahaha! I will make a crater the size of Lake Michigan on their puny planet." *Fires*

Guy On The Planet: "Watch this you space jockey." *Engages Planetary Defenses*

The Orbital Kinetic Strike is shot down. The enemy warship pulls out of orbit to get away from the defender's return fire.

Thus leaving The Enemy with no choice but to land troops. Even if there was no PDS the shock and awe won't win a war. The english thought their artillery would pound the germans back into their trenches, leaving their infantry free to lollygag through no-man's land. It failed, spectacularly. Hitler tried to bomb england into submission. That failed, spectacularly. We tried giving the germans a taste of their own medicine. It failed, and SLAGing someone just tends to harden their resolve and that of their allies. People thought that the rise of air power signaled the end of ground forces, again they were wrong.

Another problem with devestating orbital strikes is that the people on the planet know you tend to nuke whole planets, so they'll fight even harder.

I disagree with Rick's arguments for peace, they're a pleasant idea, but it'll never take place. Human beings are agressive, there will always be people sending in their grunts to invade, for any sort of reason, be that financial reasons, conquest, and/or political reasons.

Which brings us back to landing grunts on a planet (Look, if you've got FTL spaceships, you'll have the cash to carry and land grunts and pick them back up again.)

I've been reading the whole 'Space Marine Cargo Plane' debate and wondering if perhaps Space Fighters and carriers might have a viable use after all. Or rather Space Huey's.

A carrier could hold the 'fighters' and deploy them over the planet. Each craft is a one man vehicle, the pilots 'charge' through the PDS fire, open their cockpits, and fight as infantry.

In short these guys go deep into enemy territory, deploy in system, land, dismount from their vehicles and take the planet.

Civil War Cavalry Tactics in SPAAACEE!

Turbo10k said...

Okay dokey.

First off, replies!

The Scholar:

"People thought that the rise of air power signaled the end of ground forces, again they were wrong. "

Just like some people believe around here that orbital bombers can take care of anything and everything... You're right. My resolve to have ground troop invasions is partly based on the need for the omnipotent bomber to be...well...fire support.

"Each craft is a one man vehicle, the pilots 'charge' through the PDS fire"

Yep. The biggest problem with landing troops is not actually hauling them and all their wheighty equipment all the way to the battlefield, but actually putting them on the ground where they can actually do something ! As demonstrated before, drop pods are easily shot down.

"The Orbital Kinetic Strike is shot down. The enemy warship pulls out of orbit to get away from the defender's return fire."

And how are you supposed to shoot down a tugsten rod of death?

"Thus leaving The Enemy with no choice but to land troops."

Or fire another OKS from your vast stock of tungsten rods of deth. Dakka Dakka.


Geoffrey SH:
"with it diving underwater when there are no clouds."

So you are bringing up the feasability of a seaskimming, flying, submersible airshipsubcraft that carries enough cloud seed to make cumulonimbus on the spot...of course it's also armored and can fly at supersonic speeds...right?

Please forget the concept, bloggers. Pretty please?

Milo:

"If you have fusion reactors on your spacecraft, then it's not that much of a stretch to fit one on an aircraft as well."

It is.
For reasons
a)A spacraft doesn't weigh anything. It just has to lug its mass around. An aircraft has to fight against gravity every second it is in the air. This puts mass limits, which at higher ends need massive flying wing concepts, all of the latter tied down to structural limits. Remember the Spruce Goose?

b) Fusion reactors are expensive compared to comparable nuclear technologies. My nuclear thermal ramjet is hot tube in a compressor. Yours contains a full fusion reaction without any heating problems, with efficient ultra-high-speed electric engines driving some fan...

c)Depending on your tech level; fusion reactions work best in space, as gravity inteferes with the plasma density, causing lighter elements to rise and heavier ones to sink to the bottom of the reaction chamber.

d)You can probably think of some more...

Turbo10k said...

Now to the rest:

I have three scenarios for you weary and drawn out bloggers to discuss. I hope they're exciting enough...

STKONE

On far far away solar system, a small planet orbited a nearby white dwarf. This planet doesn't rotate, and has a polar cap one hemisphere and a cold, dry, dusty desert on the other side. It has only 1/5 earth gravities, a very weak magnetic field and no atmosphere to speak of. Next to the polar cap, a small domed city with a small population mines for rare heavy metals deep inside the solid mantle.
Defences are light compared to those of the major planets in the solar sytem's big planets. There's a 75MW laser on the cap side, powered by the city's fusion reactor. Mirror size is 30m diameter. It also doubles as an intersystem communication laser. Two rocket batteries can cover the whole hemisphere. On the other side, solar panels power the rocket fuel converter. A laser can be hooked up to them, but power is limited to 10MW. The city also disposed of a low tech team of police/militia to defend it against anyone creating mischief. They can move around in trucks in the cap side, but don't risk the sunny side during combat against an invading force (the sunny side is smooth and dusty, making orbital bombardment just too easy).

In space, we have the bases cargo ships. These ships are low accelerating, high capacity drones that can be fitted with bigger mirrors for their lasers if in need. Their powerplants make 60MW of electrical power. They are defended by a small patrol boat (sorry for the names) fitted with high-acceleration, high-thrust interorbit engines. It generates 150MW of electrical power, but due to its limited dV, we assume it won't try and chase down offenders.

On the other side, we have a privately-funded pirate force, 4 ships in total. We have two laser craft, a dedicated kinetic craft and a troop lander. The laser craft are comparable in combat capability to the patrol craft, but have less accel and much more delta V. The kinetics craft can launch two kinetic waves that will certainly kill any laser craft, but can risk it and send less missiles into the fray for a third shot. It also carries enough THOR rounds (1 meter, tungsten, rail launched) that we can consider their shot capacity to be infinite. The troop carrier only has a laser PDS, and no offensive weapons. Drive is comparable to that of the drone cargo ships. Onboard are some tough, power armored Mercenaries, using power armor and high-powered railguns. They can also land two vehicules armed with laser cannons.

So...Realistic enough?
Would the pirates attempt to land their troops, or would they take on the city's space forces first?

Turbo10k said...

STKTWO

We have a major interstellar war. Two empires choose to slug it out around and on top of the empires A's homeworld. Empire B has wiped out the totality of Empire A's Home Defence Fleet, but is weakened enough to not be able to face the rest of Empire A's fleet being recalled through hyperspace. Defences on Empire A's homeworld, Tellas, are uniform, very powerful, and are composed of most of what has been described to far. Gravity and atmosphere are terran, so assume we're fighting on Earth.

The major difference here is that hyperspace exists, and troops are dirt cheap. Wayyy cheaper than a hyperspace-capable combat craft. No other magitech or unobtainium though.

Troops from Empire B can drop hovertanks, fully power armored strike troops, squishy special ops squads, intermediately-armed and armored infantry armed with inexpensive rail weaponry. Empire B also fields aerospacial laser bombers, orbital laser bombers, millions of 1 meter THOR rods, several hundreds of 15 meter rods of deth to take out bunkers...the whole mess of and invading fleet.

Tellar has a uniform defence system, with hardened laser facilities, orbit-to-surface subs, semi-submerged warships (yes, submerging half a warship is very advantageous in terms of stealth and armouring), rocket trucks, cheap fighters, lots and lots of missile defences and such and such and such.

Now, won't we have a hell of a fight between landing troops and Tellas' defending forces?

Turbo10k said...

PS: It the above scenario, Empire A avoids slagging the planet because a symbolic capture of the homeworld would force the rest of Empire B to give up arms, while a vitrified homeworld means Empire A has to fight its way through every single inch of every single system with a colony planet on it...It doesn't want WWI is SPAAAAACE!

Turbo10k said...

STKTHREE

The scenario everyone doesn't want to talk about:

Surface warfare without the space bit. All the troops are already on the ground, hostile nations are next to each other, but can't slag each other due to international conventions. The techlevel is midfuture, with hypersonic laser bombers, semi-submerged and fully submerged fleets, hover tanks and missile trucks. Power armor is the norm, because as in all my settings, its as cheap as a flak jacket and steel helmet in World War II. Large populations, earth-like planet...what happens?

Cheery Reaper said...

"And how are you supposed to shoot down a tugsten rod of death?"

ZAAAPPP! Lasah Dakka.

"Thus leaving The Enemy with no choice but to land troops."

"Or fire another OKS from your vast stock of tungsten rods of deth. Dakka Dakka."

Moah Lasah Dakka!

One problem with SLAG is that it's. . . Well SLAG, congrat's you've just ticked off your enemies and made them fight even harder and have probably bought a bunch of the other (outraged) galatic empires down on your head. As well as reducing one of the few planets that can support human life to a radioactive sphere.

Who knows? Perhaps SLAG will be against treaty, after all the treaty requiring steel jacketed rounds to be used in warfare is still in effect and abided by. (One reason people abide by it is because a 5.56 is designed to wound, thus taking three people off the field)

Besides which looks better? Another habitable planet in your crown by 'glorious conquest' or a pile of radioactive slag?

In light of that, people might design 'restrained' Space BB's. Powerful enough to make the enemy miserable put not enough to make the oceans boil.

And if you break the treaty? We won't SLAG you, but we will land on your planet, and stick your head on a pike.

Geoffrey S H said...

@Turbok10

I was actually stating the capabilities of such acraft in the event of weatherctontrol not being possible at that time... I did not intend to suggest that it would carry seedlets for clouds,just weapons and defnses.

It may seem incredable, but I'm just taking this

http://en.wikipedia.org/wiki/Lun-class_ekranoplan

and this

http://www.technovelgy.com/ct/Science-Fiction-News.asp?NewsNum=1911

and putting them together, adding 500 years of technological progress and building it abit bigger. One articles I read suggested that the "ground effect" would be better the bigger the craft. The Soviet Ekranoplans, were, infact just not BIG enough.

I'll stick to subs on this blog from now on until I develop the concept further, and find someone's concept that's been done before and devloped better.

STKTHREE:

USe terrestrial assets in much the same way as they are now, but faster. fleets cross oceans quicker due to engine improvements, leaving meaning the time when they do not fight is less due to being unable to fight, and more unwilling, conserving resources etc. That DOES depnd ont eh range of misdfuture weapons- what would you have in mind there.

Lasersand counter-battery lasers egage almost instantly after the decleration of war- with laser bombers being among the first casualties of the war, for them, staying out of effective range is key to survival, getting into position would be the most important thing before the conflict.

Tanks can blitzkreig faster than ever before.

Luke said...

Turbo10k:

c)Depending on your tech level; fusion reactions work best in space, as gravity inteferes with the plasma density, causing lighter elements to rise and heavier ones to sink to the bottom of the reaction chamber.

This is not going to be an issue. The scale height of deuterium at 15 keV temperature is 74 billion meters. This means that even if you have a ten meter high fusion reaction chamber, the difference between the concentration of deuterium between the top and the bottom will be exp(-10/(74,000,000,000)), or one part in 7.4 billion. This is truly negligible. Even for boron 11, the difference in concentration at 15 keV over 10 meters is one part in 134 million (and proton-boron fusion needs much higher temperatures, which increases the scale height and decreases the concentration gradient).

And how are you supposed to shoot down a tugsten rod of death?

This is a tough one. For the guided antivehicular tungsten rods, you can take out their sensors, communications, and aerodynamic control surfaces. For the big rods used to take out fixed targets, these probably use inertial guidance and aerodynamic control surfaces to get close enough to their targets. If you take out the control surfaces, the rod will probably miss. In either case, though, the rod will impact somewhere, which could be a problem if they are being used in built up areas.

If you can hit them dead on with an interceptor, you can break them up into pieces. This not only means they probably miss their targets, but the big ones that are falling at near orbital speed will burn up a lot more - you may end up with most of it being incinerated before reaching the ground.

At 200 km range, a 10 MW laser focused through a 1 meter scope can melt-erode its way through tungsten at about 1.5 cm/s. This enables it to break the big rods up into smaller pieces that will burn up in the air (as well as take out drop pods of the smaller, guided rods). Once in the air, however, the big rods will be protected by their plasma sheaths from laser irradiation. The laser could still zap the smaller rods, though.

Milo said...

Turbo10k:

"Please forget the concept, bloggers. Pretty please?"

Don't worry, I've filed it in the same category as transforming mecha.


"A spaceraft doesn't weigh anything. It just has to lug its mass around. An aircraft has to fight against gravity every second it is in the air. This puts mass limits,"

Spacecraft need to watch their mass too. Even if you have fairly large ones, I think there is going to at least be some overlap between larger aircraft (bombers and such) and smaller spacecraft. Remember, we had nuclear-powered airplane designs already, and they almost worked, just barely failing due to factors like cost, fallout concerns, and political meddling, not because of poor aerodynamics. So even a small improvement in nuclear technology - like a fusion reactor that allows you to dispense with shielding and complicated heat turbines - would raise it from "technically possible but not really worth it" to "worth it".

Also remember that lugging a large aircraft's mass around is a lot easier when you have the power for a permanently running ramjet.

Of course, since these are aircraft, it's not just mass that matters, but also density/cross-section. If the reactor is very large for its mass, then that introduces drag concerns.

Milo said...

STKONE

"It also doubles as an intersystem communication laser."

Hehe. "They're aiming at us! THEY'RE GOING TO SHOOT! ...Oh, no wait, seems like they just wanted to talk."


"Would the pirates attempt to land their troops, or would they take on the city's space forces first?"

You stated the city has cargo craft which are not currently armed, but can be refitted for military usage. I think if I planned to get into a prolonged tangle with these people, it would be a valuable investment to attack and destroy these ships while they're weak. Depends on how difficult the refitting is compared to building a spaceship.

The generally flat and largely unsettled terrain of this planet would make it hard for surface forces to sneak around without being seen from orbit. The fact that the more important side of the planet is in permanent darkness might help with that, but infrared sensors can still see you. To land forces, I would either need to bluff the city into not recognizing them as a threat, or I would need to have at least enough space presense to prevent the city from moving their patrol boat into position to attack my troops.

Furthermore, the pirates seem to have an advantage in space power here. The kinetic craft was explicitly described as capable of defeating the patrol boat one-on-one, while the cargo ships are easy. (Yeah, there's no stealth in space, but we're pirates. We can disguise ourselves as legitimate travellers until we raise the Jolly Roger. Anyway, you did say they have cargo craft that can be converted into warcraft, not that have already been converted into warcraft.) The real problem is the defender's surface-to-orbit defenses, which are rather more of a threat.

What might be worthwhile is to land troops on the lit side and start moving them towards the city as a lure - ensuring that the main battle will take place at the edge of the enemy's surface-to-orbit defenses, rather than at their center.

Actually, wait, that shouldn't be necessary. Due to the way orbital mechanics work, if the patrol boat is in orbit at all, it has to spend exactly half its time on the poorly-defended side of the planet, no matter what. We'll nab it while it's there, even if it's in low orbit. Of course, this also means we'll spend half our time on the well-defended side, whether we want to or not. We'll use highly elliptical orbits to avoid the surface-to-orbit defenses while we're there (although on this non-atmospheric low-gravity planet, those defenses will have better than usual range).

Milo said...

STKTWO

"Now, won't we have a hell of a fight between landing troops and Tellas' defending forces?"

Well, since troops are so much cheaper than hyperspace-capable ships, it's going to be pretty hard for the invaders to muster enough forces to overwhelm the defenders, unless they have good fire support. However, while no single one of Empire B's planets could possibly afford such an invasion, Empire B has the advantage of being able to bring in a continuous stream of reinforcements from its many planets, while Empire A cannot - or at least, if it does, it'll face the same issues regarding expensive troop travel (and have to make it through Empire B's blockade of Tellas). Rich empires throughout history have often defeated poorer but well-defended factions by bleeding them to death through constant reinforcements. Of course this does mean leaving many of Empire B's planets vulnerable as they send all their forces into the fray.



STKTHREE

I can't tell how it begins, but after a while, one of the sides starts losing badly enough that it decides it would rather accept the consequences of breaking the treaty, rather than idly permitting its own destruction.

Result: one slagged Earth.

Cheery Reaper said...

Speaking of SLAG, does anyone know of any scientificaly viable ways of neutralizing nuclear radiation?

What about a Jump Point within throwing distance of the planet, thus making a surprise troop drop possible?

Luke said...

The Obsessed Scholar:

Speaking of SLAG, does anyone know of any scientificaly viable ways of neutralizing nuclear radiation?

Yes. Wait. In time, radioactive materials decay to a stable state and stop emitting radiation.

For certain kinds of radioactive materials, namely thorium, uranium and the trans-uranic elements (collectively called actinides) you can irradiate them with fast neutrons (for example in a fast reactor, or with an accelerator source, or with neutrons from a D-T fusion reaction). This causes fission, which gives you stuff which is initially more radioactive, but which decays faster (in 200 years, fission waste without the actinides is down to the radioactivity of the ore from which the fissile material was mined. With actinides in the waste, it takes tens of thousands of years).

Otherwise, you can try to trap, isolate, exclude, slow down, or speed up the radioisotopes themselves. Vitrifying waste keeps the radioisotopes trapped in a stable glass matrix, so they don't contaminate anything. If already in the environment, you can do tricks like trying to control the oxidation state of the ions so they don't flow through the groundwater, but instead remain trapped in the soil or rock surfaces. If they are already in a person, you can try to get them flushed out of the person's body, which takes medical procedures ranging from drinking a lot of beer (for tritium exposure) to chemical chelatants (for various radioactive heavy metals). If they are not yet in a person, you can try to keep them out by various medical procedures (such as eating a lot of iodine, for potential radioiodide exposure) or behavioral methods (protective clothing, hand washing, not going in to contaminated areas, radiation monitoring after returning from contaminated areas, etc.).

Thucydides said...

If both sides have near parity in technology, any ground war will be a long, drawn out slugfest as the various systems cancel each other out. The World Wars of the last century should demonstrate that point, and WW III (the Cold War) never became "hot" due to the realization that the end result would be the destruction of the homeland. I imagine that we could degenerate to WWI level combat where troops hide in metamaterial shielded trenches and go over the top after a barrage is delivered by one of the surviving orbital systems....

The ultimate lesson of the three World Wars of the 20th century was that logistics is the winning card, the German Empire and Triple Entente had nowhere near the industrial capacity of the Triple Alliance nations and their subject Empires, the Axis was overwhelmed by Allied war production and the USSR was crushed by defense spending that amounted to an estimated 25% of its GDP (which was already only a fraction of the American GDP).

The lesson of the small wars of the second half of the 20th century and the start of the 21rst is that willpower is the key to victory; any powers that engage in this kind of conflict is signing up for decades of effort. Logistically, any Western power could carry out these wars blindfolded (indeed, consider that despite fighting two wars and engaging in a global game of "whack a mole" with Jihadis, there is no rationing or any sort of deprivation or sacrifice in the United States or any of the Allied nations). The political will to carry out the war to a successful conclusion is questionable, however, and leads to a series of "on and off" wars (OIF is round three of the Persian Gulf War, and a putative conflict with Iran would be round four.)

Cheery Reaper said...

I thought of something.

The old missile vs Naval Gunfire angle.

Missiles, can do a lot things, naval shells can't, missiles are expensive, so you can only have so many.

Shells, you can lob thousands onto someone's beach for cheap.

So, suppose I try to nuke the other guy with missiles. He PDS's them. I try it with an asteroid (IIRC Atomic Rocket's has a section on how SLAG by asteroid is easier said than done) I switch to Tungsten Rods of Death and Lasers.

While they make him miserable, they don't win the war, however they supress the enemy long enough that I can land men. . . Oh, wait. But if he's suppressed why not nuke him?

His defenses are inter-woven. mutually, and self-supporting of course.

So in effect my TRoD/Laser bombardment provides cover fire for my squishy Marines as they cower in their drop pods (Although I'm still toying with idea of the 'Space Huey's').

The enemy won't waste effort on my TRoD's he'll keep his head down and keep an eye out for things like asteroids and nuclear missiles.

He'll probably leave the drop pods alone as well, after all he's got guerilla's and Imperial Troops waiting, again he'll concentrate on asteroids and nukes.

So while Orbital Strikes will be important and can provide cover fire so my Marines can land. It can't kill a planet.

Voila, Space Navy, Marines and Conventional Slug Fests. And reasons to have them.

Geoffrey S H said...

You sure you want to call them marines Sholar? Atomic Rockets suggests the term Espatier.

I think I get what you'e aiming at- why waste millions on risking your prescious shiny spacecraft and anti-planetary WMD's when you can send somewhat cheaper troops in. Of course, those troops might one day become so well-equipped in the ensuing arms race, and spacecraft and WMDs become far cheaper in comparison that a slugfest from orbit becomes the only option again. Some sort of military future history could be developed out of this.

Do you have a setting in mind? or have you not crafted one?

Geoffrey S H said...

Thucydides:

Trenches are, after all a feature of the battlefield and have been around since ancient times...

Cheery Reaper said...

"You sure you want to call them marines Sholar? Atomic Rockets suggests the term Espatier."

Do you happen to have a link to the entry? :) Does Atomic Rockets have anything on Surface Warfare BTW?

"Do you have a setting in mind? or have you not crafted one?"

Actually, yes :D. It's still in the rough stages, but so far I've got a Hive Mind, a Republic, an Empire and an alien empire. And (possibly) Psychics

Milo said...

The hive mind: aliens or transhumans?

Geoffrey S H said...

http://www.projectrho.com/rocket/rocket3k.html- Espatiers link.

Alas, I do not think there is anything on planetary warfare, but there is actually quite abit for navies on the net.

Cheery Reaper said...

Milo" "The hive mind: aliens or transhumans?"

Aliens, for the most part they leave other people alone except for sporadic trade, mostly because both humans and hive mind find each other extremely hard to comprehend.



Geoffrey "http://www.projectrho.com/rocket/rocket3k.html- Espatiers link.

Alas, I do not think there is anything on planetary warfare, but there is actually quite abit for navies on the net."

Thanks for the link! :)

Yeah, I'm beginning to think that this entry is probably the first of it's kind.

Turbo10k said...

"Another habitable planet in your crown by 'glorious conquest' or a pile of radioactive slag?"

Is kinetic bombardment radioactive?! Well, maybe if you use depleted uranium...

"That DOES depnd ont eh range of misdfuture weapons- what would you have in mind there. "

Tomahawk strike ranges for railguns, with any cruise missile having near intercontinental ability as can be carried easily by drop planes. I'll try not to have a push-a-button-fire-from-next-hemisphere type of battles, as I believe that laser PDS eliminate extremely long range shots.

"with laser bombers being among the first casualties of the war,"

Being so fast and having such long range, I think they can affort launching later in the war, only when ground defences are found to have a blind spot. Remember, due to their height advantage, laser bombers can fire much further than ground lasers are limited to, due to the horizon...

"If you take out the control surfaces, the rod will probably miss"

Not if my expensive TRoDs are fitted with ablative control surfaces ie burn themselves into shape through combustion. I invented it. As future proposed rockets are near entirely made up of a column of solid fuel, why can't we ignite a bit of that fuel so it shapes itself into a flap?
Also, the rod can be intelligent enough to eject pieces which are deformed to keep a nice aerodynamic shape.

"Spacecraft need to watch their mass too"

They can afford to go slower.

"Depends on how difficult the refitting is compared to building a spaceship."

As easy as exchanging a cargo pod for a pod with a big mirror.

"We can disguise ourselves as legitimate travellers"

Or not. The pirates fully well know that even if they are detected a zillion miles away, they only endanger themselves if they want to. They can't be chased and orbital defences can't afford long range surface to ship missiles...at least not this one.


More coming.

Milo said...

Turbo10k:

"They can afford to go slower." [regarding difficulty of fusion reactors on aircraft vs spacecraft]

What now?

Spacecraft speeds are measured in kilometers per second. 1 Rick (about 2.89 km/s) is considered a space-snail's pace.

By comparison, aircraft think that the speed of sound (about 0.343 km/s) is fast. Only hypersonic (well beyond supersonic) aircraft would even get into the low end of spacecraft speeds.

And spacecraft have to struggle to push themselves forward the hard way, by expelling reaction mass, while aircraft engines are much cheaper on account of having something external to push against.

The only think spacecraft have going for them is that they have no drag, so they can afford to accelerate gradually over long periods, while aircraft need to continuously produce high thrust throughout their trip. However since they're airbreathing and can therefore operate at low "exhaust velocities" without this trashing their "delta-vee", they can actually manage to get more more acceleration from a given amount of power than spacecraft can.


"Or not. The pirates fully well know that even if they are detected a zillion miles away, they only endanger themselves if they want to. They can't be chased and orbital defences can't afford long range surface to ship missiles...at least not this one."

The stealth is in order to ambush the cargo craft before they're able to refit, and more generally in order to give the defenders less time to "dig in" when they realize they're under attack. The stealth isn't in order to prevent being preemptively attacked, since as you say that couldn't happen anyway.

Turbo10k said...

Milo:
Oh... okay.
I just don't see how the dome city citizens can "dig in". You're not going to hit it, because it's your prize. They can't hide in it because if they do, they can't defend themselves and will have to give up when you ransom their air supply. Can't build any more weapons either (no weapon factories) and there's no point in moving your lasers or rocket launchers as such big installations are easy to see...
I also came from the assumption that a dedicated kinetic craft will be pretty obvious due to its hard-to-hide rail guns and high density.

To what I haven't replied to yet:

"Well, since troops are so much cheaper than hyperspace-capable ships, it's going to be pretty hard for the invaders to muster enough forces to overwhelm the defenders,"

What I meant is that any hyperspace-capable ships can go back and forth from Empire B's homeplanet to bring more troops unhindered. Even if they have low individual capacity, the number of trips they can do provides leverage.

Oh wait. Too many tech assumptions. How fast can a hyperspace-capable ship go? How far away are Tellas and Empire B's homeplanet? How much can each troop carrier carry? How many trips can it do during the invasion? What is the ratio of troops carried (minus those killed during drop) to that of Tellus' defenders? Sorry for that...

"Of course this does mean leaving many of Empire B's planets vulnerable as they send all their forces into the fray."

Worse, they are under pressure as there's a full fleet returning from the colonies...Empire B's objective is to conquer Tellas before said fleet returns, and keep the Emperor and the government hostage during peace negotiations. If they're not fast enough, they'll be caught in a crossfire between the defenders and the counter-attackers... Tellas has to bide its time.

"So while Orbital Strikes will be important and can provide cover fire so my Marines can land. It can't kill a planet."

Err...I thought you had to take cover from lasers because you can shoot down kinetics, and Orbital Strikes can slag planets while lasers are just a menace to ground troops...

"Trenches are, after all a feature of the battlefield and have been around since ancient times..."

Cos trenches are made of dirt, and dirt is well, dirt cheap...

On Espatiers:

In my setting, China, Brazil, Russia, France are the leading countries militarily. New names for Space Marines will probably be quite different if sprouts from a different language. I'll keep Espacier for Franch BTW. Espace is Space in franch.

Turbo10k said...

9 more posts...

Cheery Reaper said...

Tubro10K: "Err...I thought you had to take cover from lasers because you can shoot down kinetics, and Orbital Strikes can slag planets while lasers are just a menace to ground troops..."

Good point, although it would depend on the size of the TRoD. How big would it have to be to gain Planet-Killing Status? And how could you have it realistically where said TroD wasn't big enough to cause a nuclear winter and a small earthquake?

Although I think the tactis might still apply. He lasers my missiles and TroDs' he lasers my asteroid.

I'm forced to resort to throwing lasers at him. He keeps his head (the PDS) down for important stuff like TRoD's, nukes, and asteroids. My lasers provide cover fire for my landing craft and drop pods.

Luke said...

Turbo10k:

Not if my expensive TRoDs are fitted with ablative control surfaces ie burn themselves into shape through combustion. I invented it. As future proposed rockets are near entirely made up of a column of solid fuel, why can't we ignite a bit of that fuel so it shapes itself into a flap?

Wait, are you seriously suggesting making a re-entry device expected to be exposed to shock heated plasma at temperatures exceeding that of the sun's surface out of rocket fuel? Regardless, if I have a laser that can burn through the entire thickness of the rod in seconds, or a direct impact by a counter-missile kinetic, your ablative control surfaces will simply be gone. There will be nothing left to ablate.

Also, the rod can be intelligent enough to eject pieces which are deformed to keep a nice aerodynamic shape.

Are you suggesting it does this after being burned into segments, or smashed into segments by a kinetic? Remember, at orbital velocities the impactor can expect to lose the first meter or so to ablation by the radiated heat of the shock wave (which, again, will be hotter than the surface of the sun). So a three-meter rod, broken into three one-meter pieces, will not reach the ground. Nor will it reach the ground if the damage causes it to assume an orientation with a non-zero angle of attack with respect to the on-rushing air.

Now the antivehicular kinetics will not evaporate after they have been aerobraked to about 2 to 2.5 km/s. If you hit them early, they will be entirely consumed since they are only about 1 meter long. If you hit them later - depending on how you hit them, a damaged but functional rod may still be able to impact its target, but only if its sensors and attitude control are still intact.

Milo said...

Turbo10k:


STKONE

"I just don't see how the dome city citizens can "dig in"."

Preparing. Calling in reserve forces, having a combat plan in mind rather than panicking when you're suddenly coming under fire, warning the civilian population. Taking up strategic locations and formations if there are any.

But ultimately, better safe than sorry.


"I also came from the assumption that a dedicated kinetic craft will be pretty obvious due to its hard-to-hide rail guns and high density."

Even so, it takes a lot better sensors to recognize individual components (and can't they be hidden behind a shutter?) than to detect an exhaust flare.

If density is really that much higher than other ships, a Whipple shield is a nice way of making yourself appear larger than you are.



STKTWO

"Even if they have low individual capacity, the number of trips they can do provides leverage."

Ah. Expensive ships, but cheap fuel and short travel times.

In this case, since Empire B's forces have to be brought in gradually in small groups, you need to be able to keep them safe until they get reinforcements. Tellas's defenders would love to divide and conquer your occupation forces, surrounding and destroying each unit as it lands rather than facing them all at once.

Thus Empire B's tactics would revolve around capturing a defensible beachhead region as staging area for the rest of the invasion.

Milo said...

By the way, here's something to look at if you're looking for exotic vehicle tactics.



The obsessed Scholar:

"How big would it have to be to gain Planet-Killing Status?"

According to the boom table, our well-known extinction-event level impact was about 100 teratons of TNT (half a yottajoule). Killing off all higher life (including tenacious species like humans) needs a little higher than that. If you don't want to take chances, circa 100 petatons of TNT would suffice to blow off the atmosphere and/or boil off the oceans, which surely constitutes adequate planet-killing under any militarily useful definition. Meanwhile, 200 megatons of TNT gives you a Krakatoa-like explosion, which is enough to lower global temperatures for a year, with ensuing famines and unpleasantness, sufficient that an explosion in Indonesia can make a painter in Norway feel a great disturbance in the Force, as if millions of voices suddenly cried out in terror and were suddenly silenced (no, really) - however, all this will merely make your victims angry rather than significantly damage them. 50 megatons of TNT - not that much below Krakatoa - gives you a Tsar Bomba, which is really rather localized. Actually shattering a planet would require tens of zettatons, but that's really overkill.

This suggest you are likely to need megaRicks (1% c) to slag with ship-sized missiles, or else you need to haul huge asteroids. Not easy. For comparison, our current worldwide energy production is a little over 10 gigatons of TNT per year.

Cheaper slagging can be accomplished if you selectively target all major cities with city-killer bombs, without causing global devastation.

Milo said...

Another thought: vehicles, not just troopers, need to be adapted to conditions on the target planet if they're meant to be used in an invasion. This is particularly an issue for airplanes, where differing gravities and air densities are going to seriously affect your aerodynamics.

Rick said...

Yikes, coming up on 700.

The term espatiers for spaceborne troops was coined, so far as I know, by world builder Tyge Sjostrand in an email discussion with me. The analogy is to our use of 'marines,' also from French. (As are most military terms: European geopolitics of the 2nd millennium in a nutshell.)

Curiously, no one seems to ever use 'spaceborne,' in spite of the familiar analogy of airborne. The maritime trope strikes again.

I differ on one point, I forget now whose. Mass destruction weapons are cheaper than troops - it costs less all-round effort to deliver one nuke warhead as to deliver one espatier.

So if you are invading with espatiers, the implication is that you're willing to risk losing rather than simply nuke/smack the place.

Thucydides said...

I like the term "Exeine" to describe ships troops. It is less nationalistic (what do German or Chinese derived space navies and colonies call their troops?), and more importantly, I invented it ;)

Thinking back to Heinlein, his space navy did apparently have the means to slag planets (I believe he actually said "crack them open like an egg" in Starship Troopers, but memory fails and my copy is buried somewhere), but the Federation maintains an MI and associated troop formations in order to calibrate the use of force for politically useful purposes. Since a political settlement with the Bugs isn't in the cards based on my reading of Heinlein, the main reason the war rages on is planet busters are pretty rare and expensive.

To interpolate a bit from this reading of Heinlein, a planet buster is probably a Nova bomb or massive KKV capable of accelerating to a good fraction of c. The carrier vessel will be quite large, and will need some sort of task force to provide an escort and service the beast. Perhaps it doubles as a carrier for "battle riders" (Johnny Rico's small company sized troop carriers might benefit from this sort of basing), so provides the ultimate covering force for the invaders; defeat the MI and you should be prepared to be blown out of space...

Cheery Reaper said...

Rick: "Curiously, no one seems to ever use 'spaceborne,' in spite of the familiar analogy of airborne. The maritime trope strikes again."

Indeed, I was toying with calling my ground troops Fallschirmjagers

"I differ on one point, I forget now whose. Mass destruction weapons are cheaper than troops - it costs less all-round effort to deliver one nuke warhead as to deliver one espatier."

Turbo10K I believe, however a nuke costs several million, it takes a lot of time to put together, and then you have to load it up and send a plane off. An expensive plane with an expensive pilot, both of which cost a lot of money and the pilot takes a lot of time and money to train.

"So if you are invading with espatiers, the implication is that you're willing to risk losing rather than simply nuke/smack the place."

I'd think the implication would be that I'm tired of having my expensive (and small in number) nukes shot of the sky. And in space I think that cost of building a nuke and then loading it onto a ship and then hauling it off to war would be even more expensive than it is now. That and the crew and the ship itself also add to the cost.

Raymond said...

For terminology, I'm partial to a term from a Tony Daniel short story: Skyfaller.

Milo said...

The obsessed Scholar:

"Fallschirmjagers"

Fall screen hunters? Sorry, I don't speak German, only Dutch, so I'm trying to match the words. What did you derive the term from?


"Turbo10K I believe, however a nuke costs several million, it takes a lot of time to put together, and then you have to load it up and send a plane off. An expensive plane with an expensive pilot, both of which cost a lot of money and the pilot takes a lot of time and money to train."

Is "several million" really that large compared to the price of training and equipping a battalion of soldiers?

The plane and pilot costs are irrelevant because you would beed to pay the same costs for troop transports (in fact, troop transports cost more because they need life support systems rather than just a bomb bay).

Troops do have the advantage that they're (somewhat) reusable, whereas nukes are not. However, between combat casualties whittling them down, and the fact that you won't be fighting all that many wars in quick succession, this isn't too much of an advantage.

Turbo10k said...

Sooo...

Going backwards (as usual):

"The plane and pilot costs are irrelevant because you would beed to pay the same costs for troop transports"

Speaking of pilots...
In my setting, mainly due to the large influence of 'Battle Angel Alita', most military personnel takes on a brain in jar form. There's the space-going brain in an armored box personnel, and the ground equivalent of brain in superhuman clone.

This is because:
-I hate the life support argument when dealing with drones
-Gives me big gee thrusts in combat
-Gives me Superman to play with on the battlefield.

Pilot costs and troop training costs, or even the cost of maintaining the pilot and keeping the troops alive, are therefore irrelevant. You can train a pilot safely back home, then carry a bag of jelly to be connected to his plane in orbit before battle. Troops are grown on-the-spot (inside the future troop carriers) during the inbound journey towards the battlefield, and troops are connected before drop. I don't have magical brain memory transmission stuff (lol ref to 'The Dun hills of Earth'), so death is still and issue, but my brains in jar don't die once you pull the plug, only several days later...
Just to give my take on future interstellar combat.

"I'd think the implication would be that I'm tired of having my expensive (and small in number) nukes shot of the sky."

Not going to happen. I'll rase the defences with my plentiful and cheap TRoDs, then keep the nukes for destroying the now vulnerable and defenceless area. I'm digging for allegories...okay. If the planet I'm at war with is a dangerous criminal, orbital bombardment with TRoDs is equivalent to tackling him and putting him behind bars, and the nukes are the lethal injection...(to all u 'Muriccans).
Nukes are only kept until needed.

"defeat the MI and you should be prepared to be blown out of space..."

Now THAT'S an ultimatum. Like a sci-fi Damocles' Sword.

"This is particularly an issue for airplanes, where differing gravities and air densities are going to seriously affect your aerodynamics."

I'll only bring airplanes in to fight high-priority targets. High priority targets are nearly always Earth-like, thick-atmosphered planets with big populations. Anything less and I'm better off with lasing from orbit. After all, we're only bringing in airplanes because of the thick air and cloud.

"Tellas's defenders would love to divide and conquer your occupation forces, surrounding and destroying each unit as it lands rather than facing them all at once"

Except if you amass them in high orbit, then land them all at once...

PS: I like using the abbreviation 'TRoDs' as it reminds me of trodding on someone, crushing underfoot.

PS: Orbital bombardment is actually quite...clean. Big bangs and destruction, but nowhere near the century-long contamination of nukes.

PS: I don't like slagging. It is highly inefficient in purpose once you have destroyed the enemy's infrastructure. War is used to submit one's will upon another. Completely annihalating someone else is pointless, in the measure that there's no need to weaken him any further than to the point of becoming harmless...

PS: Nuclear hot-rod thermal jets work whatever the air is composed of. Their efficiency decreases with density (right?), but they have to work less because the plane doesn't have much to push through. Air density would only affect a plane if lift is difficult to achieve under a certain speed (hypersonic waveriders come under pressure here), or you need VTOL capabilities.

Turbo10k said...

Happy 700 posts!

Milo said...

Turbo10k:

"Except if you amass them in high orbit, then land them all at once..."

You can't do that, because that requires having some ship in orbit capable of housing all troops at once. You don't have hyperspace-capable ships that large.

I guess you could build a bunch of non-hyperspace ships, then carry them over with multiple runs of a hyperspace tugboat.

But non-hyperspace ships are still considerably more expensive than ground fortresses.


"PS: Orbital bombardment is actually quite...clean. Big bangs and destruction, but nowhere near the century-long contamination of nukes."

With kinetic bomardment (and anything else that makes a big boom without nukes), your biggest environmental worry would be atmospheric dust - which is what killed off the dinosaurs. See also: nuclear winter, volcanic winter.

This requires some pretty big booms to be an issue.


"Air density would only affect a plane if lift is difficult to achieve under a certain speed"

Yeah, that's what I was thinking. Lower air density means you need bigger wings to stay up at a given speed. And I'd expect that having wings much bigger than you actually need would introduce stalling and drag problems.

Cheery Reaper said...

Milo: "This requires some pretty big booms to be an issue."

So would it be realistic to have a battleship that could fire TRoD's big enough to make the enemy miserable, but not were they were big enough to cause a nuclear winter?

Turbo10K: "Not going to happen. I'll rase the defences with my plentiful and cheap TRoDs, then keep the nukes for destroying the now vulnerable and defenceless area. I'm digging for allegories...okay. If the planet I'm at war with is a dangerous criminal, orbital bombardment with TRoDs is equivalent to tackling him and putting him behind bars, and the nukes are the lethal injection...(to all u 'Muriccans).
Nukes are only kept until needed."

And I laser down your TRoD's?

Milo said...

"So would it be realistic to have a battleship that could fire TRoD's big enough to make the enemy miserable, but not were they were big enough to cause a nuclear winter?"

How miserable are we talking about? 20-ish kilotons of TNT gives you a Hiroshima/Nagasaki level bombing. You can fit 10000 of those into a single Krakatoa. Even if you make your city-flattening bombs 10 times bigger, and want the total energy to stay under 10% of Krakatoa's (I don't know what the actual limit on nuclear winter would look like), then that still allows you to obliterate 100 cities. I think that counts as making the enemy miserable.

Actual dust levels will depend on various conditions, like the type of soil you're hitting, the amount of wind available to carry it across the planet, and such. It's actually fairly likely that a meteor impact of a given energy will cause less dust than a volcanic explosion of the same energy.

Another idea is to hit the sea next to you target, hard enough to cause a tsunami, but not hard enough to plow into the seafloor throw up dust. That's gonna hurt.

Cheery Reaper said...

Milo: "Fall screen hunters? Sorry, I don't speak German, only Dutch, so I'm trying to match the words. What did you derive the term from?"

The German Airborne in WWII. After all we see 'Panzers' in mil sci-fi. :)

Milo said...

Ah, I see now. "Fallscreen" is a nice term for a parachute.

Turbo10k said...

Okay dokey:

Germans in space...humm.

"You can't do that, because that requires having some ship in orbit capable of housing all troops at once."

Totally forgot about that. In my setting, the only 'hyperspace-capable' part of a 'hyperspace-capable ship' is the generator which translates it from realspace to hyperspace. Tugs, warships, anything can move from orbital defenders to being able to cross lightyears with a simple refit. I didn't want to go into all of these details because all I wanted to do was create a situation where the space-bourne ground forces only suffer a small numerical handicap compared to their land-legged equivalents...Take it for done, I want to concentrate on the massive landing and ground-warfare-with-orbital-support part.

"And I laser down your TRoD's?"

Damn laser vs missiles argument. Either lasers can blast a ton of SNCF per second, or my TRoDs are overwhelming death from above. Oi! Think realistic, if we have TRoDs AND lasers PDS in the same setting, we'd think they'd be balanced enough to be considered being put in use on a battlefield...

More later.

Turbo10k said...

Continued;

"TRoD's big enough to make the enemy miserable, but not were they were big enough to cause a nuclear winter? "

The only two kinds of TRoDs I'd use are the 1 meter anti-matériel ones (tank formations, ships at sea) and the 3 meter plough-all-the-way down bunker buster ones. The latter are nowhere near creating nuclear winter.

For massive destruction, as in I need to level that mountain now, I'd pummel the surrounding laser defences (saturation tactics would take over of they are very effective) until I can get a clear drop for the nuke. One nuclear bomb isn't going to make a nuclear winter.

"Lower air density means you need bigger wings to stay up at a given speed. And I'd expect that having wings much bigger than you actually need would introduce stalling and drag problems."

Bigger wings or higher speed. Bigger wings would be needed if you cannot easily or don't want to exceed mach 1.6. This is the case for the commercial lifting body designs of 2050, or if you only have access to turbofans...

For example, the F-104 Starfighter had very thin, very small wings. That was okay because it went very fast (mach 2.6 for the last, never flown version) and didn't care about low-speed maneouverability.

I just don't know how that will work out for waveriders, as they don't provide lift using wings, but by riding on shockwaves...

By the way, no one has replied to the aerobraking ship drop concept, or how Marines not being so squishy and difficult to transport because they're brains in jars...
:(

Cheery Reaper said...

Tubro10K "By the way, no one has replied to the aerobraking ship drop concept, or how Marines not being so squishy and difficult to transport because they're brains in jars...
:("

There, there. :) I like the Aerobraking ship idea, I mean you get the whole 'Huey in SPAAACCCEE!' angle and you get the tough Aerobraker Pilot-Jock type swaggering around bars and hooking up with hot women. :)

Gotta go, more latter.

Cheery Reaper said...

So you can have TRoD's without SLAG?

Okay, then, but why would the enemy warship only carry three meter TROD's?

Why would the enemy not load the magazines with rounds 50 meters in diameter and 150 meters long?

Byron said...

Overkill. You won't ever need something that big, and it takes a lot of mass to haul it around. If you do need to kill something really big, a nuke is a much better option.
Plus, something that big is a really good target for defenses, making it unlikely to get through intact.

Luke said...

The obsessed Scholar:

At 50 meters diameter and 150 meters long, a tungsten impactor would mass 5.7 million tons. That's a bit heavy for most plausible mid-future spacecraft to carry around, and would take a lot of work to de-orbit.

So ask yourself how much mass you have to haul around in the form of tungsten to blast a city, compared to how much mass a nuke would take. Which one will be cheaper to de-orbit? A 20 kT boom in the form of de-orbited tunsgten would require about 2,000 tons of the stuff. Are the nukes in your setting really that massive?

Cheery Reaper said...

Byron: "Overkill. You won't ever need something that big, and it takes a lot of mass to haul it around. If you do need to kill something really big, a nuke is a much better option.
Plus, something that big is a really good target for defenses, making it unlikely to get through intact."

Good to see you back Byron! :) Thanks!





Luke: "At 50 meters diameter and 150 meters long, a tungsten impactor would mass 5.7 million tons. That's a bit heavy for most plausible mid-future spacecraft to carry around, and would take a lot of work to de-orbit.

"So ask yourself how much mass you have to haul around in the form of tungsten to blast a city, compared to how much mass a nuke would take. Which one will be cheaper to de-orbit? A 20 kT boom in the form of de-orbited tunsgten would require about 2,000 tons of the stuff. Are the nukes in your setting really that massive?"

Nope, the nukes aren't anywhere near that size and weight :) Thanks!

So would it be realistic to have a BB that carried several thousand 1 ton TROD's? And is it believable to have BB's and Frigates that are powerful enough to throw up a lot of mud, level an area the size of WWII Hiroshima, but not so powerful that they could make an ocean boil? (Sorry if that's a dumb question :) Thanks!

Milo said...

At 50 meters diameter and 150 meters long, your impactor is a ship-sized missile. A large-ship-sized missile. It would be treated more as a hellburner than as ammunition. It would also be a strategic weapon, given that even at low meteoric velocities it still does near-Krakatoa level damage.

Milo said...

The obsessed Scholar:

"So would it be realistic to have a BB that carried several thousand 1 ton TROD's?"

Well, several thousand 1 ton projectiles would mass several thousand tons (duh). So the question is if you can afford to build ships big enough to be able to carry several thousand tons in their ammunition hold alone.


"And is it believable to have BB's and Frigates that are powerful enough to throw up a lot of mud, level an area the size of WWII Hiroshima, but not so powerful that they could make an ocean boil?"

Boiling the oceans takes around 100 petatons of TNT. The largest nuclear bomb ever built, the Tsar Bomba, was 50 megatons of TNT. That's a scale factor of some 2 billion. You could drop a Tsar Bomba on every city on Earth and still fall short of 2% of the Chicxulub impact energy.

Boiling the oceans is not a concern, at all, unless you deliberately set out to do so. Environmental devastation from nuclear winter is a much more realistic concern, and will come into play at much lower energy levels. I only brought ocean-boiling up at all because you asked about planet-killing, and boiling the oceans would be a good way of making sure that you really killed everything (Chicxulub level impacts still leave survivors, as we can attest).

Byron said...

I've been busy at Rocketverse, and with a move.
To level an area the size of Hiroshima, it would take a weapon of about the same power as Hiroshima. That's almost nothing as nukes go, and not too much for strategic kinetics.

Thucydides said...

If you are looking to really whack someone without using gigaton sized bombs, anti-matter or improbable torch drives, consider that a solar sail making a close pass by the sun can accelerate (theoretically) to 13 psol (.13 c).

Now a skydiver weapon like this will still need a fairly sophisticated flight profile, for example, using the sail to kill orbital velocity relative to the sun, furling and moving behind a heat shiled during the dive (or approaching the Sun edge on to minimize heat and thrust to the sail), then separating from the heat shild and rapidly deploying or re-orienting to turn face on to the Sun to accelerate...

Add the mass of the weapon and then calculate how many Ricks of energy you will have to play with.

Rick said...

700+ comments ... wow!

I'm not a fan of massive kinetics as WMD, since nukes seem easier, but note that in a setting with colonies and extensive trade, throwing heavy loads through space at high velocities is the shipping industry's bread and butter. So slinging a lot of mass is not necessarily a problem. (Though several million tons is a LOT of mass.)

Milo said...

If nothing else, kinetic bombardment using commercially available mass drivers could be a larger-scale equivalent of the AK-47 and IEDs - a cheap but effective weapon available to relative peasants and ragtag rebels, even though well-prepared militaries can do better. Because a nuclear IED is likely to be a tad difficult to make, even in a setting with very cheap space travel.

For more imperial forces, there's also the terror weapon value, since if you really want to make it clear that insubordination will not be tolerated, then hurling one big (Chicxulub size) asteroid sends a clearer message than carpeting the planet with a bunch of smaller nukes. Make it really clear to guerillas that you're not interested in playing whack-a-mole.

Overall though, I think people underestimate the difficulty of moving a multi-teraton (actual mass, not TNT equivalent) rock, even at Hohmann delta-vees. Just that the rocks are there doesn't mean you can use them against your enemies, any more than armies on Earth are in the habit of tossing (conveniently nearby) mountains at each other. Chicxulub slagging, no matter how cool it would be, is something you can only do if you have massive amounts of resources to spare. It's one of those plainly wasteful things that powerful empires do to show that anyone who can't afford to be equally wasteful isn't in the same league as them.

Byron said...

I really think that kinetics will be divided into two types: tactical and strategic. Tactical is what has been discussed, with small weapons that attack tanks or bunkers. Strategic kinetics range from destroying cities to slagging planets. However, nukes are cheaper and more reliable.

KraKon said...

In fact, the tactical/strategic boundary is not defined here by mass or destructiveness but rather by time to target.

The one meter rods that target tanks and ships arrive under half an hour, decelerate to 2000m/s then search their targets.

The three meter+ bunker busters take a few hours to deorbit, and hit at near orbital speeds. They would be strategic as they can level cities in laregr versions.

In that case, I don't know where to put nukes. They deorbit like tactical rods, and hit at even lower speeds, yet they do more damage than the strategic rods...

Hurling a Chicxulub is a big operation, but it's also a deadman-switch type all-or-nothing weapon. Deadman-switch because even if the invading fleet is obliterated, you've still got a massive asteroid pointed at you. All or nothing because you can't target it, and if it hits, you die, if it misses, there's no way the invader is turning it around...

As for aerobraking ships, they're not space Heuys, they are pinecone interplanetary ships loaded with drop pods that fall as deep as they can into the atmosphere at interplanetary speeds, opens its flaps, drops the pods then exits into an long elliptic orbit.

Byron said...

No, that's not the case. Both will take similar amounts of time to deorbit. They are launched by a ship in orbit, and go inward for about half an orbit, then hit. As is the case now, tactical and strategic will be defined by use, which will in turn determine destructiveness. Tactical nukes are designed for use on the battlefield, and are small compared to strategic nukes, which target either hard targets or are meant to take out multiple targets at once, which results in large yields.
Actually, most people think that strategic nukes are aimed at cities. This is wrong. They're aimed at military targets, which just happen to be in cities. For example, St. Louis, where I used to live, was the number three target city for the Soviets during the Cold War. This was because of the McDonnell-Douglas plant and the military records center. It wasn't because of the Arch. However, if we have precision kinetics, we don't need to take out the entire city to get a target. This allows us to use "tactical" kinetics on a "strategic" target. Thus, I'm going to update my definition. Strategic kinetics are anything that has an energy yield comparable to or greater than 50 tons, that being the largest conventional bomb available, and overkill for anything that's not a city. Anything larger is likely for terror bombing.
Nukes are definitely strategic, and a Chicxulub is completely pointless. If you can redirect it, you can initiate nuclear winter far more cheaply.
And for dropping ships, just use the Starship Troopers method.

Milo said...

Re: the tactical vs strategic distinction, a "strategic" weapon connotes to me that you're in a sense bypassing normal combat.

War can be thought of as taking place on a strategic map, where you do things like moving knights towards an enemy castle, and then when they encounter enemy forces, you zoom in to a tactical map "minigame" where you attempt to take down individual walls and turrets of the castle using entirely different rules.

A strategic weapon is when you decide you're not in the mood for the minigame, and decide to just raze the castle in one go directly from the strategic map.

Byron said...

That's a decent way of describing it. Maybe I should change my terms. I'm trying to draw a distinction between the use of kinetics as modern conventional bombs are used and as nukes are used. Both can be used both strategically and tactically, but not in the same way. I'm postulating that kinetics will replace conventional bombs, but not nukes.

Milo said...

Well, today we have a concept of "tactical nuke", although none have yet been used in battle. I think there is a little overlap in strength between the smallest tactical nukes and the largest conventional bombs.

Whether kinetics can efficiently replace nukes depends on how many Ricks you're capable of. Given that 100 km/s still only gives you 1.2 kiloRicks, though, which is very good but still inferior to typical nukes, nuke-level kinetics aren't likely to appear until pretty late in the future history.

Replacing conventional bombs, of course, just requires 1 Rick. You get more than that for free just from orbiting.

Do remember that small kinetics may be acceleratable to much larger speeds than actual spaceships. But that does still mean producing energy comparable to a nuclear explosion in your gun barrel...

Luke said...

A projectile entering the troposphere at 100 km/s will generate dynamic pressures of around 6 GPa. This exceeds the ultimate strength of tungsten by about a factor of 4. The listed value for tungsten is for tensile rather than compressive strength, but you could still expect to mechanical failure at these speeds, and buckling of long rods at much lower speeds.

A rule of thumb that works as a crude first approximation far better than I would expect is that the temperature in Celsius of the shock front pushed ahead of a hypersonic projectile is roughly equal to the velocity in meters per second. I expect this breaks down above 10 km/s or so, but if it holds up to 100 km/s you are looking at the front end being exposed to something on the order of 5 TW/m^2 of radiated heat. This will erode the front end very rapidly, removing a considerable fraction of the rod before it could impact.

KraKon said...

Or how about a hyper-cavitating 100km/s rod? You shoot a ramhead made of diamond (can support 10GPa? Maybe cheaper alternatives) ahead of the rod, creating a much less dense plasma channel that the following tungsten rod can go through...

Luke said...

KraKon:

Or how about a hyper-cavitating 100km/s rod? You shoot a ramhead made of diamond (can support 10GPa? Maybe cheaper alternatives) ahead of the rod, creating a much less dense plasma channel that the following tungsten rod can go through..

If you do that, the diamond will experience a greater pressure than the tungsten, and hence a greater force. With a presumably lower mass, this means that the diamond will be pushed back faster than the tungsten rod. Either it will collide with the rod (transferring the stress into the tungsten since now the air pushes the diamond which pushes the tungsten), or it will be deflected slightly and miss the tungsten rod, exposing the tungsten to the full force of the atmosphere's drag.

You also have the issue that from the mesosphere to the ground is about 120 km, so it will take a bit over a second for a 100 km/s rod to traverse that part of the atmosphere where it will be exposed to the radiated heat from its plasma sheath. At 100,000 K, this will melt-erode the rod at a rate of about 200 m/s, so the first 200 meters of your rod will be removed before it can smack the ground.

Mangaka2170 said...

So, how about a rod made of diamond? While this assumes that artificial diamond can be just as strong as natural diamond and manufactured much more cheaply, it is a possibility.

Luke said...

Diamond is less dense than tungsten, so a diamond rod would need to be about ten times as long as a tungsten rod of the same mass and frontal area. This gives a much longer span for buckling. Plus, diamond is brittle, so any failure would shatter the whole rod. The diamond is a bit better behaved at tungsten under the irradiation of its 100,000 K shock front - it would only lose the front 80 meters or so of its length rather than the front 200 meters.

Raymond said...

Luke:

Unless my grasp of reentry mechanics is completely screwed up, wouldn't there be a cooler plasma layer between the bow shock and the tip of the reentry vehicle? Ablative heat shields for modern spacecraft aren't a meter thick, and they see reentry velocities of 10 km/s when returning from, say, Lunar missions. Is it just a difference in design from what we're talking about here?

Luke said...

Raymond:

You are completely correct. For a well-designed re-entry nose shape, this shields the spacecraft from heat transfer by conduction from the plasma layer to the spacecraft skin. However, there is still radiation from the plasma layer to consider. The cool insulating layer of air is transparent to this radiation (mostly - in the mid and far infrared you can have a fair amount of absorption, and in the vacuum ultraviolet and soft x-ray parts of the spectrum you also get absorption - I am neglecting this, since it turns the problem from a straghtforward calculation of radiation from a surface in thermal equilibrium to a complicated problem of radiation diffusion. Complete transparency is probably a reasonable approximation over most of the temperature range we are considering, but the figures I mention might decrease a little, especially at the higher temperatures). The figures I gave are for the radiated heat from a 100,000 K surface.

Raymond said...

Luke:

I'm a little confused where the 5 TW/m^2 actually comes from, considering the energy of a 100 km/s object is "only" 5 GJ/kg.

Luke said...

Raymond:

There is a rough approximation that the temperature of the shock front of hypersonic objects in Earth's atmosphere in K is about equal to the velocity in m/s. It is unknown if this continues to be approximately useful at velocities of 100 km/s, but lacking a better estimate, I chose to use it to see what would happen. At 100 km/s, it predicts a shock temperature of 100,000 K. Using the relation for black body radiation, the radiated intensity is 5.67E-8 W/(m^2 K^4) * (temperature)^4, which comes out to 5.67E12 W/m^2.

Raymond said...

I'm not sure if we should be using that approximation, then, as the emitted power seems an order of mag or two higher than I'd expect.

I got a tad over 5.1 TW/m^2 when I did the blackbody equation, but then I used an emissivity of 0.9 instead of 1. Is a high-temperature plasma considered (nearly) perfectly emissive? And now that I think of it, wouldn't the majority of the emissions be in the hard UV to X-ray range?

Luke said...

Plasma is a good approximation of a black body at wavelengths longer than the wavelength corresponding to the plasma frequency. For fully ionized air at sea level density, this corresponds to a wavelength of about 350 nm. The plasma will be transparent to shorter wavelengths.

Unfortunately for our estimate, the density of the air at the shock could be quite a bit different than that of the surrounding air.

Raymond said...

If the emissions from the plasma are substantially in the UV range, wouldn't they also be absorbed by the air in front of the bow shock, too? And what effect would this have on the size and density of the shock? (I suspect it would enlarge the shock zone and thus the plasma channel, lowering the equilibrium temperature, but I could easily be out to lunch.)

Rick said...

As usual the technical aspect is way above my physics pay grade.

I love the working definition of 'strategic' weapons. Curiously enough the usage developed historically from cavalry, but with much the same idea. American Civil War experience (if not earlier) led to the concept of 'strategical cavalry' striking deep into the rear to disrupt the enemy without directly engaging his forces.

Has much been written about the fact that both air forces and armored forces were basically developed by ex-cavalrymen?

Luke said...

Raymond:

High energy photons emitted from a shock into unshocked air can have ... interesting ... behavior. If the radiation is not too extreme you just get a rounding off of the shock jump, but if you have intense radiation the speed of radiation diffusion can overtake that of the shock and you get an isothermal expanding radiation front. It all gets very complicated - but for our purposes we do not need to worry about it. Simply because if it happens, then the energy scales are such that solid matter is essentially irrelevant and our impactors will detonate high in the atmosphere. A lot of interesting physics was done on radiation-driven thermal transport in the vicinity of high energy shocks in the mid to late 40's and 50's. You can probably figure out why.

Cheery Reaper said...

Rick: "I love the working definition of 'strategic' weapons. Curiously enough the usage developed historically from cavalry, but with much the same idea. American Civil War experience (if not earlier) led to the concept of 'strategical cavalry' striking deep into the rear to disrupt the enemy without directly engaging his forces.

Has much been written about the fact that both air forces and armored forces were basically developed by ex-cavalrymen?"

Hmm, good question. And I don't think much has been written.

Hey, I just thought of something, it seems like we've been assuming the PDS is going to be fairly static.

However, would it not be Plausible Mid-future tech to have mobile PDS systems? Or would these systems be so expensive and large that it's better to bury them under a mountain?

Milo said...

I figure the size of point defense depends on what you're trying to defend against. You're likely to see vehicle-mounted point defense, though, both in order to defend the vehicle itself (for large vehicles) and as the sole purpose of smaller vehicles attached to a formation. If I were outfitting an infantry unit, I'd like to include some point defense on my APC - if it'll fit.

KraKon said...

Same ol' habit:

"If I were outfitting an infantry unit, I'd like to include some point defense on my APC - if it'll fit."

Sure you could fit a minilaser onto your APC, but I don't think point defences are easily scalable. Go too small and they're pointless, too big and they become offensive weapons...

"Has much been written about the fact that both air forces and armored forces were basically developed by ex-cavalrymen"

They're just the same tactics developed hundreds on years ago but applied to different situations, theatres and technological levels. Even the Romans used units designed to go around the bulk of the enemy army and attack the command posts at the rear...

"Hmm, good question. And I don't think much has been written. "

Just to show you that they can be easily re-invented and applied the same way time and again.

"However, would it not be Plausible Mid-future tech to have mobile PDS systems? Or would these systems be so expensive and large that it's better to bury them under a mountain?"

As for truck sized PDS, we have HUMVEEs equipped with parabolic mirrors. A laser defence system can easily replace the Phalanx CIWS on the future all-electric destroyers.

The massive mountain-side lasers are offensive weapons. They attack invading ships in orbit. They are so well defended because they are easily found the moment the fire, so need to withstand counter-fire.

"radiation-driven thermal transport in the vicinity of high energy shocks in the mid to late 40's and 50's. You can probably figure out why."

Not me. Microwave jets? Nuclear blast waveriders?!

"(I suspect it would enlarge the shock zone and thus the plasma channel, lowering the equilibrium temperature,"

I thought it was obvious that the diamond ramhead would be sent out a few seconds (or hundredths of a second, depending on speeds involved) before the following tungsten rod. The diamond ramhead is supposed to carve a channel of plasma ahead of the rod, and plasma being much less dense than air, it would be easier to go through. Worried about the plasmas' heat? Coat the rod in SNCF, or diamond again if not too expensive.

Of course, again, I don't know how wide and long the plamsa channel of a 50cm diameter ramhead makes, at the speeds we're talking about; nor how long it lasts...

"5.1 TW/m^2"

WOW. So a rod designed to go through that can easily bask in the exhaust plume of torch drive fusion rockets.

Milo said...

KraKon:

"The diamond ramhead is supposed to carve a channel of plasma ahead of the rod, and plasma being much less dense than air, it would be easier to go through."

Wouldn't the ramhead experience more resistance, though, and consequently slow down and not stay a few seconds ahead of the rod?

Luke said...

KraKon:

I thought it was obvious that the diamond ramhead would be sent out a few seconds (or hundredths of a second, depending on speeds involved) before the following tungsten rod. The diamond ramhead is supposed to carve a channel of plasma ahead of the rod, and plasma being much less dense than air, it would be easier to go through.

The low density wake behind a hypervelocity projectile will not last long - only about the projectile's radius divided by the speed of sound. This means that the tungsten rod will need to be following less than a distance of about (initial projectile's diameter)*(projectile's velocity)/(speed of sound). For our example, this is less than about 300 diameters behind the precursor projectile. If the precursor projectile is one meter in radius, the tungsten rod will need to follow within about 300 meters and about 3 microseconds behind the precursor. At that close distance, the increased drag experienced by the precursor will smack it back into the rod.

WOW. So a rod designed to go through that can easily bask in the exhaust plume of torch drive fusion rockets.

The converse, of course, is that no rod can be designed to withstand this kind of irradiance.

Thucydides said...

Point defense weapons only make sense if the object being protected is considered a "high value" target. Tanks are getting point defense systems like "Trophy", "Arena" or "Drozd", partly because tanks are far more expensive than APC/IPC class vehicles and partly because tanks tend to draw much more attention to themselves (and thus much more enemy fire). Surface warships can cost more than a billion dollars, and since capital ships are few in number the loss of one or more would be devastating both operationally and financially.

Spaceships would seem to fall under the same definition as capital ships, and are imbued with a lot of potential protection by their nature (the available energy in the drive system, kinetic energy due to their speed and the ability to house lasers of stupendous range). Groundpounders are almost by definition low value targets (even SOF types), so point defense is usually a non starter unless you have magitech or make some very unusual opening assumptions in your story/game setting.

Troops will benefit from area defenses, so things like laser or microwave weapons that can protect a zone from incoming projectiles, aircraft etc. are being researched in the here and now.

Milo said...

I guess area defences is what I meant. I see it as a "point" defense because it's meant to protect a single unit on the march from those things you mentioned, not a wide swath of territory (but also not a single soldier).

Geoffrey S H said...

@ Thucydides- perhaps a smal pds unit (with the appropriate developments in miniturization) might be part of a heavy weapons groups (known in the UK as a "GUN GROUP" that includes the heavy machine guns, etc that support an infantry unit.

I.e.: haning back beind an advancing unit and supporting them, only with both "defebnsive" and offensive weaponry. It wouldn't threaten incoming missiles of kinetic rods of course, but it might be useful against rpg type weapons...

Geoffrey S H said...

* sp small
*sp defensive
*sp hanging

tkinias said...

Jumping in after 750 posts...

Nearly all the discussion here seems to have been about planetary assault and defence. I'm rather more interested in the purely planetary aspect, where all parties have spaceflight but originate from the same planet.

For the setting I'm working on (mid future, no FTL, Earth-centric, geopolitics dominated by Great Powers and their clients), ground warfare has three very distinct forms:

1) Peacekeeping operations: Great Powers operating against criminals/insurgents. Great Power forces operate under UN auspices with tight rules of engagement. Lines between police and military activity are never as clear as authorities try to make them.

2) Limited Great Power conflict on Earth: Great Power conflicts are limited by the impossibility and undesirability of full conquest and occupation and by a strong regime of international law (no nukes in atmosphere, no chem or bio weapons, no targeting civilians, etc.) An important wrinkle is that autonomous weapons are illegal: no hunter-killer robots, landmines, or autonomous submunitions.

3) Extraplanetary "ground" warfare: Great Powers fighting in, around, and over colonies and settlements on other planets in the solar system (all non-"shirt sleeve").

I'm especially interested in people's ideas about #2: What would warfare look like between two highly professional Great Power armies, if both are committed to minimizing civilian casualties and strictly following international law?

Thucydides said...

I'm especially interested in people's ideas about #2: What would warfare look like between two highly professional Great Power armies, if both are committed to minimizing civilian casualties and strictly following international law?

I think that horse bolted a long time ago. Once the gloves are off, all aspects of the enemy society and economy are targets since any/all of them can be used to prosecute the war effort. This principle was applied as far back as we have records (ancient Greeks liked to devastate the croplands of their enemies, Sherman's "March to the sea" was an updated version of the medieval chevauchées, Germans used unrestricted submarine warfare in WWI and the allies used massive bombing raids in WWII).

Modern concepts include cyber warfare to disrupt critical systems and cripple the enemy economy, and suicide bombers and other terrorist attacks on targets in the homeland (most terrorist organizations are funded/influenced or perhaps even controlled directly by State actors as a form of asymmetric power projection). High tech militaries can bring fire support from another continent, so disproportionate damage can happen anywhere (B-2's flying from Montana to hit targets in Kandahar today. Railgun artillery on ships with ranges of 300+ km tomorrow...)

So look at surface combat as being bloody and dangerous to everyone.

Byron said...

But where do you draw the line on autonomous weapons? Is a shell fuse autonomous? What about a bunch of normal submunitions? A GPS-guided bomb? A Longbow Hellfire? I would think a smart submunition is less dangerous than a normal artillery shell. It's far more likely to only hit tanks, not cars.

And B-2s fly from Missouri, not Montana.

Mangaka2170 said...

Yeah, if we're deploying ordnance from Montana that means IBCMs, which can hit any stationary target on the planet's surface from any point on the surface or near orbit (like around the Moon).

If tensions are high enough between nations (which are social constructs, as opposed to countries which are simply political fictions), a war between two post-industrial powers on the same planet could end with total annihilation of the other being the eventual objective (like if India and Pakistan, at a tech level approximately 500 years from now, were to start up another shooting war). Although the science in the show is sketchy at best, the anime Mobile Suit Gundam SEED does a good job of describing the political atmosphere such a war might have.

Byron said...

I've done more thinking on the "limited great power war" scenario, and come to some conclusions:
There will have to be ridiculously strict rules, rigorously adhered to. The entire concept reminds me of dueling. It will have to be set up so that it's impossible for either side to win by too great of a margin, and the entire thing enforced by the rest of the great powers. Any war will have to be between two and only two powers, with very limited objectives and restrictions on weapons. The problem is that if one side sees that they're going to lose, the leadership might order the use of nukes anyway. There has to be a mechanism to make sure that wars are ended before that point.
The entire thing reminds me of war before the French Revolution. The real problem starts when someone breaks the rules. That will almost certainly lead to nuclear exchange.

Rick said...

Three quarters around the track! And welcome to another new commenter.

This last turn in the discussion brings me back to the point of the 'peace' thread: The people who said 100 years ago that war was obsolete may merely have been premature.

In a nutshell, great power war either has to be so restricted that it becomes a formalized game, not Deathmatch 2250, or, if really fought to the hilt, it goes nuclear and both sides lose.

The latter is certainly possible, but not as a social institution, because it selects against itself.

tkinias said...

Byron:
But where do you draw the line on autonomous weapons? Is a shell fuse autonomous? What about a bunch of normal submunitions? A GPS-guided bomb? A Longbow Hellfire? I would think a smart submunition is less dangerous than a normal artillery shell. It's far more likely to only hit tanks, not cars.

That's indeed a tricky point. Basically (in this setting) people had a strong, visceral this is wrong reaction to the widespread employment of hunter/killer drones in twenty-first-century (largely asymmetric) warfare.

Rules don't, BTW, necessarily have to make sense: look at today's ban on chemical weapons, which holds that killing someone quickly and painlessly with a (hypothetical) chemical agent would be a war crime but burning him alive isn't.

Basically, in this case, the law requires a human being to "pull the trigger". I don't see indirect artillery fire as being very important, basically because the volume of fire necessary to overcome effective point defences means that you're just saturating an area rather than going after a specific target and that area fire would itself be considered a war crime... but you do have a paradox where firing an unguided howitzer round against an enemy tank would be technically legal but more likely to cause unintended damage than dropping an illegal autonomous seeker.

Cheery Reaper said...

One thing about nukes. The idea of a nuclear holocaust might not be an
issue if the two superpowers or space empires are using fusion
weapons. What with the (very) low yield.

It also seems to me that we're leaving the PDS SDI system out of the
equation. Conventional warfare could very well be in the cards if the
two sides are using fusion weapons and SDI.

Cheery Reaper said...

BTW what sort of radar/detection systems will the PDS be using?

Byron said...

Only if SDI stops all the nukes. There are more ways than ICBMs to deliver nukes, and many are harder to stop. Cruise missiles are a good example.

tkinias said...
This comment has been removed by the author.
tkinias said...

Also Byron:
There will have to be ridiculously strict rules, rigorously adhered to. The entire concept reminds me of dueling. It will have to be set up so that it's impossible for either side to win by too great of a margin, and the entire thing enforced by the rest of the great powers.

The only way a war "goes total" is when (a) one side refuses to accept anything but unconditional surrender or (b) one side refuses to accept that it's beat.

Basically, one or both parties to the war have to think that the peace would be worse than the war. Maybe you think that conquest would be worse than death (for ideological reasons, or because the conqueror really would annihilate/enslave/whatever you). Maybe you think that the enemy is so beyond the pale that they must be destroyed utterly.

But if you reasonably believe that you can survive the process you will want peace. Think of it as a plea-bargain. If you know you can't beat the charges, you'd prefer a plea bargain than a trial conviction, right? Unless it the prosecutor refuses to make a deal...

Any war will have to be between two and only two powers, with very limited objectives and restrictions on weapons.

I'm not sure I understand why "only two powers"...

In fact, I think that war is more likely to be total in a bipolar world. In a multipolar world, there are other powers to act as a check on abuses by the victor (making a conditional surrender more attractive). There are also reasons for keeping your enemy around: what happens to your relationship with Prussia if you completely absorb the Austrian Empire?

(cont.)

tkinias said...

(part II)

The problem is that if one side sees that they're going to lose, the leadership might order the use of nukes anyway. There has to be a mechanism to make sure that wars are ended before that point.

The only way the leadership should even be tempted to use nukes before admitting defeat (assuming that using nukes isn't accepted as normal behaviour) is if defeat means losing their status. We have established a precedent that the victor gets to rework the government of the vanquished. That's actually (historically speaking) abnormal.

Let's say Freedonia is at war with Insanistan. Insanistan has nukes. If Freedonia has made it clear that a Freedonian victory means trying the Insanistani leadership as war criminals, nobody should be shocked that the Insanistani leaders would try to nuke Freedonia before admitting defeat.

If, on the other hand, the Freedonians were very clear that they would accept a 50% stake in the Insanistani lithium mines, payment of a big indemnity, etc., as peace conditions, and would in return help ensure that the current Insanistani leadership didn't get strung up by its own people after the war... it's a whole different ball game.

The entire thing reminds me of war before the French Revolution.

That's pretty much what I had in mind.

Byron said...

That's pretty much my point about the nukes. The problem I have with the scenario is that it requires that everyone be reasonable. And plus, there can't be much point in fighting if you can't win much.
Only two powers is because the have to be the others to serve as moderators. In some ways, the only way it'd really work is if there was a formal treaty spelling out how the members would settle disputes that went beyond what diplomats would settle. The two involved would have a "war" while the other powers agree to moderate.

Cheery Reaper said...

Would laser armed hyper-sonic drones solve the problem of cruise missiles?

BTW no one answered me on what kind of rader/detection systems the PDS will be using.

tkinias said...

Byron:

The problem I have with the scenario is that it requires that everyone be reasonable.

Maybe the difference is that I think people mostly act in what they perceive to be their own interest.

I guess I should make explicit that this is one of my underlying assumptions: if people's behavior appears totally unreasonable, then either you are misunderstanding their motivations or they are misunderstanding their situation.

Only two powers is because the[re] have to be the others to serve as moderators.

Ah, right. I had misread you as saying it could occur in the context of a two-power system (like US-USSR in the Cold War) rather than a multi-Great Power geopolitical system.

In some ways, the only way it'd really work is if there was a formal treaty spelling out how the members would settle disputes that went beyond what diplomats would settle.[...]

I'm not sure it has to be a formal treaty; it could be the international version of common law.

Y'know, I'm reminded of how aircraft hijacking has basically disappeared. It worked as a terror/resistance tactic for a long time because there was the basic expectation that if the hostages played along, they stood a good chance of surviving. After 2001, a hijack attempt will be "total war" because it is impossible for hostages to trust that "conditional surrender" will save their lives. OTOH, we do have maritime piracy because ship crews believe it's better to surrender than die, and by and large the crews do get ransomed rather than murdered. There's no formal treaty governing it, but there is a set of expectations that leads to pirates and ship crews not "going total". If analogous expectations obtain in the realm of Great Power relations, then war will tend to be self-limiting.

This does not, of course, prevent war from becoming total ‒ or prevent pirates from murdering the crew ‒ but it makes possible long periods of limited war or piracy.

Thucydides said...

As wars drag on, the governments tend to demonize the enemy more in order to maintain the will to fight on their side. As well, the stakes increase as more blood and treasure is expended, if only to make the eventual victory seem to be worth the cost.

The American Civil War is a fairly clear cut example, escalating from "States Rights" to Abolition, with the victory conditions rising as well (negotiated settlement might have been achieved early on, but towards the end, the unconditional surrender of the Confederacy and a pretty harsh occupation was all that could be expected. Circumstances limited the damage here, Lee essentially quashed any plans for a Confederate insurgency and Lincoln was not planning a harsh occupation [but his death probably made the occupation harder than it would have been]).

A similar dynamic evolved during World War Two; it is possible that a series of negotiated settlements could have taken place and that period would have been regarded as a spate of medium sized wars between various Empires, rather than the various conflicts merging into a global conflict with unconditional surrender for the vanquished.

Major Power war in the nuclear age will probably take place at the margins, small wars and insurgencies sponsored by the major powers to shore up their sphere of influence, while working on economic and diplomatic edges that can be exploited as well (think of China and the recent threat of withholding rare earth elements to Japan unless the Japanese released a Chinese fishing captain).

Milo said...

The obsessed Scholar:

"One thing about nukes. The idea of a nuclear holocaust might not be an issue if the two superpowers or space empires are using fusion weapons. What with the (very) low yield."

Why would the powers be deliberately using weak weapons when strong ones are available?

Also, please remember that while we do not yet have pure fusion weapons, we did already test a few weapons that released the majority of their energy through fusion. One of them was the Tsar Bomba.



Thucydides:

"As wars drag on, the governments tend to demonize the enemy more in order to maintain the will to fight on their side. As well, the stakes increase as more blood and treasure is expended, if only to make the eventual victory seem to be worth the cost."

In a real limited war, you sould simply give up and admit defeat when you perceived that the blood and treasure costs are starting to outweigh your original justification for fighting.

However the real problem is that war for resources isn't so socially acceptable these days. People today strongly believe in the idea that a war must be fought for the pursuit of some "just" ideal. This doesn't mean that wars will never start over money in practice, but to pass public opinion, the politicians have to have the skill to spin it in a way that makes it sound like it's being done for idealistic reasons. For example, the US has often been accused of invading Iraq for the oil, but the official party line was always "protecting the US from Saddam's WMDs" or "liberating the oppressed and downtrodden citizens of Iraq". And idealistically motivated wars are exactly the ones that demonize the enemy. You never say "Hey, those people across the border are all nice and happy, but I feel a moral imperative to go bomb them to pieces.".

Now if you could find a way to reverse these attitudes...

Cheery Reaper said...

Milo: "Why would the powers be deliberately using weak weapons when strong ones are available?"

Sorry, I meant to say low *radiation* yields.

Would that work?

Anonymous said...

As for warfare on Earth in the Plausible Midfuture (TM), you might want to consider remote controlled drones; not just aircraft, but sea and ground combat drones as well. These would be controled from heavily fortified and armed bases, ships, or moble platforms. Wars would mostly be fought by proxy forces and nations; the Great Powers providing resources, RC mechanized weapon systems, monitary support, ect. This is an extrapolation of current trends, but it seems plausible.

Ferrell

Milo said...

The obsessed Scholar:

"Sorry, I meant to say low *radiation* yields.

Would that work?"


First, that wouldn't really help MAD. If your city is flattened, then your city is flattened even if it's not irradiated. Radiation just adds insult to injury.

Second, the physics doesn't work that way. Simple version: for any fusion bomb you can make, I can make a mixed fusion-fission bomb that makes a much bigger boom for the same price.

Rick said...

Also, even with a pure fusion tech, the difficult and costly part will still be triggering the explosion in the first place. Once you can do that, adding yield is relatively straightforward and cheap, whether it is pure fusion or fission-fusion.

And as someone noted a few posts up, while radiation (like chemical weapons) has an extra sinister reputation - rooted in the traditional detestation of poison - it is the flash and bang, fire and blast, that wipes out cities.

I also agree with the point that unconditional surrender is historically the exception. Remember that even World War I ended with an armistice.

Cheery Reaper said...

The more we debate this the more issues we come up with :D. It's not unbelieveable to create a PDS that can shoot down missiles, thus preventing MAD, it's not unbelieveable to have a sophisticated enough PDS to laser and/or missile and/or rail-gun down an asteroid. And yet the problem of having a PDS that's good enough to give one a reason to have 'Spaceborne' units and getting any significant amount of those troops to the ground is still running around.

The tactic I mentioned earlier seems to be the only one that would get any amount of men to the surface alive. Although that too has it's difficulties (The British in WWI thought their artillery would force the germans to keep their heads down long enough for their men to cross no-man's land. And if you can force the enemy to keep his head down with TROD's why not throw nukes while he's distracted? The idea that the enemy would stick his head out long enough to laser an incoming nuke is. . . problematic at best)

Perhaps you could throw conventional missiles at him in diversion. Thus he diverts his PDS taking out the missiles allowing your troops to land unharmed. . . But that raises the question of how come those troops don't just plant a nuclear warhead and fly back to their transport, where they detonate said nuclear warhead from orbit. Then you tell the enemy on the planet that you will detonate the other warheads you planted unless he surrenders.

Blasted Nuclear weapons. They ruined everything.

The final solution I can think of is to have a heavily armored space assault ship. The forward bays carry a couple of divisions of ground units, the middile bays hold air units, the rear bays hold 'wet' naval units.

This behemoth descends through the atmosphere and through the PDS fire, and lands on a beach, deploys the units and then lifts back into orbit. Or it might not even do that it could just be towed through space by a pair of Frigates and then once it's on the ground it just deploys it's troops and sits there to rust away. Although I'm sure something like that (Let's call it an assault lander) raises a myriad of problems and questions of it's own.

Byron said...

I'm going to introduce you to the kinetic weapons calculator. The problem with the assault ship is that it simply can't be armored enough. If we take a warhead the size of that on a SCUD missile (985 kg, 44 cm radius) and put it stationary in the path of your ship moving at 6 km/s (reentry profile) it will go through 316.5 m of carbon or 533 m of titanium. Actual penetration won't be that great, but it still shows armor to be pointless, except against fragments and very small rounds.
A ship like you propose is far to large to make effective decoys for, and is incredibly valuable. The enemy will use everything they have to stop it, and likely succeed. Plus, while you're in the plasma sheath, you can't see what's going on to defend yourself. Point defense might help some, but it won't be enough when there are dozens of weapons coming in at once.

Thucydides said...

I say we take off and nuke the entire site from orbit. It's the only way to be sure.

Cheery Reaper said...

Well. . . That's the end of that idea.

Anyone else?

Raymond said...

We keep talking about a ground assault against a unified planet. Isn't it more likely that any planet capable of covering its surface area with PDS weapons is too densely populated to remain unified? And if one has a) a large ally on the ground and b) enough lasers in orbit to function as a good PDS against nukes, what does that war look like?

Cheery Reaper said...

Hmmm, I don't know.

I do wonder why you'd have lasers in orbit? Why not under a really dense mountain? I don't have a problem with that idea (It's pretty cool) I'm just wondering what you're reasons are.

BTW we've been assuming the PDS covers the whole planet.

However, he who tries to protect everything loses everything. So my thinking is the PDS would cover the places where you'd evacuate your civilians, and the most important of your major population, resource and industrial centers.

Thus leaving large swathes of territory where you can land without being butchered on the way down.

As for nuking the planet into submission via those uncovered swathes of ground. . . Why would you nuke the place you're going to land? Besides the enemy probably took fall-out into account and took measures.

Thus we have a reason(s) to land troops and occupy a planet. And we have a viable way to land troops without complicated magitech solutions and equally complicated tactics.

Raymond said...

I meant the lasers in orbit of the attacking (/supporting) constellation.

Byron said...

I still think that you far underestimate the difficulty of a ground campaign. While some zones (those around important areas) will be far heavier than others, that doesn't mean that those others are left open. I'd leave a few missiles in the wilderness to make sure the enemy didn't try to land troops there. If he did, you shoot at the biggest landers, which likely contain something valuable. I've already made analysis of the mass of a battalion for transport purposes, and I will again state that it is not really possible to transport enough people to seize a defended world. If 6700 tons is a large spacecraft, then we're looking at one large spacecraft per 700-ish soldiers. My earlier numbers came to about 10 tons per man. Or, for a more precise breakdown:
Men and equipment, no supplies, per man: 5.126 tons
Shipboard supplies, per man per month: 93 kg
Combat supplies, per man per day: 75 kg.
Peacekeeping supplies, per man per day: 55 kg.
I know that the combat supplies dropped, but that's assuming that any vehicles in use don't require fuel to be dropped to them. I've recently gotten a copy of James Dunnigan's How to Make War, from whence the new supply numbers came.
The same books states that it takes between 200 and 2000 soldiers per million enemy civilians to maintain order depending on the population's reaction to your presence. This is between .02 and .2 percent.
I'm going to attempt the logistics of moving troops across interplanetary distances again using rocketverse as an example. Cepha will be invading Fram Dirio. Transit time is 1.5 months (35 km/s). If we budget for 8 months of peacekeeping (until the next synodic windown), the total per battalion will be
That means that for each soldier deployed it will take 18.612 tons, of which 13.3 tons is peacekeeping supplies. The comes to between 37.2 and 3.72 kilograms per person on the planet. In other words, to subdue a hostile planet for 8 months, you have to move half the weight of the occupants in stuff. For Fram Dirio, that comes to between 2000 and 20000 troops. The total deployment would require 37,224 to 372,240 tons. That's a lot, and it doesn't even count combat forces or supplies. (Yes, you can use combat troops for peacekeeping, but you'll probably need more men for combat than peacekeeping. Iraq was an aberration.) The numbers are rapidly spiraling out of hand. If you use brutal tactics you might cut the total troop requirements, but I can't see anything below 200. If you're nice and they like you, that'll work. If you're not nice, they won't like you no matter what.
The point of all these numbers is simple. The act of moving ground combat troops over interplanetary distances is very difficult. To fight your way from the wilderness to a target would require a lot of forces. Very soon, it's far easier to just divert an asteroid than to move the troops.

Cheery Reaper said...

Does that change if you've got a very fast FTL and a million worlds under your control?

As for landing, while a few missiles might inflict some casualties, it won't stop me. While someone's family is going to very sad that his lander got hit by a missile, I won't stop simply because I lost a squad or even a platoon of men on the way down.

Byron said...

It does change with FTL, and might work, but that's not the environment I was doing the math for. And it might not destroy your assault to loose a few platoons, but there will be a certain number of missiles that will blunt an assault. First, the missiles won't be aimed at infantry barges, but at the landers carrying tanks, artillery, and aircraft. The loss of those during a light infantry assault will have a disproportionate effect. Second, how much in the way of losses can a unit stand. Units sometimes become combat-ineffective with as little as 10% losses. Other figures suggest that as much as 30% is required. Either way, the defender doesn't have to shoot down all, or even most, of the landers. Even a few will seriously hamper combat power. There is a tradeoff in the size of the pods. Individual pods will get the most people down, but those that do get down will be the least effective, because of the lack of heavy weapons (which would be the first targets) and the scattering of units. Larger pods require fewer missiles to disrupt the drop, but the units are more effective on the ground. Also, the support forces are less vulnerable to specific targeting, although they would still be somewhat visible due to the greater mass of the pods.
Now that your units are down, you still have to march them overland to the target. That's not as easy as it sounds. This discussion was had earlier, but I'll repeat it. First, the units might well be 500 km from the target. Second, they won't have tactical orbital fire support during the assault (no lasers). Third, even unopposed, they will probably move no faster than 25 km/hr. Fourth, the defenders will be dug in, significantly increasing their combat power. Fifth, the defenders will outnumber the attackers by a good bit, as they don't have to be shipped from another planet.
I'm not going to bet on ground assaults.

KraKon said...

Same old habit:

"Units sometimes become combat-ineffective with as little as 10% losses. Other figures suggest that as much as 30% is required."

How large are these units? A five man squad? Losing a man or two would then be catastrophic.
But if we're sending down 30 men at a time, they can change their tactics and use their fewer numbers with a proportionate, not catastrophic loss of performance.
At even larger scales, as in the whole army losing 20% of its numbers...well, that's what an army is designed for-retain effectiveness in the face of losses.

"Does that change if you've got a very fast FTL and a million worlds under your control?"

My third scenario, TKTHREE. Needs a lot of magitech though, so much that I belive they would change the nature of conflict so much it would be unrecognizable (from a tactical point of view. You're still trying to land troops and neutralize defences..)

"The same books states that it takes between 200 and 2000 soldiers per million enemy civilians to maintain order depending on the population's reaction to your presence. This is between .02 and .2 percent. "

In the case you have humans wearing sleeves and holding rifles. I think that can be considerable reduced with the use of near-direct control robots and low orbit support using lasers. If each robot replaces half a human, with a friendly face per ten robots...we can go down to .004 percent.

"Very soon, it's far easier to just divert an asteroid than to move the troops."

Now that's striking as unnecessary. You already have nukes, thousands of TRoDs and lasers hanging overhead. Using "I'm moving Ganymed your way" would be just as effective "I'm going to melt your cities" when you have that much firepower for much less cost and much much less time (available the moment you enter orbit).

"Third, even unopposed, they will probably move no faster than 25 km/hr."

Not if we use a Space Huey concept, an aerobraking drop, a water landing...cities are vulnerable to coastal attacks!

"BTW we've been assuming the PDS covers the whole planet. "

Impossible for a single laser, and you'd need an Intercontinental missile with a maximum reaction time for 30mins for the other PDS.

"And if one has a) a large ally on the ground and b) enough lasers in orbit to function as a good PDS against nukes, what does that war look like?"

Refer to my rant against political sterility.

Nukes slow down just like drop pods, so if you can shoot down all and any nukes, you can shoot down all and any drop pods.

One more thing for ALL of you to think about:

Bombing from space NEVER wins a war. You can lose it, if all your ships are lost, can make a draw if you can't destroy enough defences, but against a determined opponent, all you can do is slag or leave. And slagging is not winning in my book.

Byron said...

My numbers are for divisons. That may not be the best example, but it still isn't going to take killing everyone to render the unit ineffective.
I'm ignoring magitech, as it's, well, magical. We can't run numbers.
I would think that occupation is where humans are the most important. Drones might work for open-field warfare, but for "boots on the ground" men are needed. And drones probably require almost as much in the way of supplies. I'll be generous and say half as much as a man. The total mass is the same.
I'm not advocating astroid strikes, meerly pointing out that it's really, really, really hard to move enough troops to pacify a hostile and heavily-populated world.
And any method of moving faster exposes you to more lethal defenses. Shooting at things on the ground is the hardest targeting environment there is. We have lots of fire and forget anti-ship and anti-air missiles, but not as many for use on ground targets.
Nukes don't have to slow down. They work just fine going fast. Bombing from space won't win a war. But having total control of low orbit, along with a few troops on the ground, can. If the ships have lasers, they become like artillery. Very accurate, instant-response artillery.

Cheery Reaper said...

I was wondering. In the PMF would the medical tech be advanced enough that you could take stuff that remove the nuclear rads from your body as fast as they came in?

I know, it's the firey blast that kills you, but the fact is people fear the radiation most of all.

Also, Byron, why does going faster make you easier to track? And easier to nail?

Byron said...

If you're referring to my comments about the vehicle pods being trackable, that's not exactly what I meant. What I was getting at was that the pod's mass will be visible to ground-based sensors (discussed earlier), and that tanks are more dense than troops, so you should be able to see the support weapon pods. Those pods will be prime targets for the defender, as aircraft are significantly more valuable and rare than troops.
Radiation isn't some sort of disease. It's actual, physical damage. The means by which it kills living things is cellular-level damage (knocking out pieces of DNA and protein). Computers, which work on the same scale, are also vulnerable. Mass objects aren't. Thus, it's very difficult to repair it, particularly in real time. It might happen, but I think it's more likely that genetic engineering will be used.

Rick said...

The Galactic Empire can afford planetary invasions, even under conservative assumptions about the difficulty (which I tend to agree with).

But how it gets to BE the Galactic Empire is more problematic. A trade empire need rule no planets directly, at least in theory, but you can gin up scenarios where from time to time it needs to invade one.

Raymond said...

Byron:

"For Fram Dirio, that comes to between 2000 and 20000 troops. The total deployment would require 37,224 to 372,240 tons. "

You give those numbers as if they were impossible. They are not. It may be a large undertaking, but don't rule it out.

Also, the total supplies per soldier can quite possibly be brought down using local resources - and compared to planetbound logistics, with interplanetary invasion there's much more incentive to live off the land, so to speak.

Byron said...

Yes, but a lot of that is ammo and spare parts. You can't get those off the land. Maybe you could ship a factory or modify one, but I don't see any significant first-year savings.
It may not be an insurmountable amount of stuff. However, it would take 1525.5 space shuttle launches to put that in orbit, or 312.8 Saturn Vs. That's a lot of stuff. Even if you trim it by half, it's still a lot. Even if launching is two orders of magnitude cheaper, that's still a lot of stuff. Just for the occupation. You still have to consider the actual combat.

Raymond said...

Actually, isn't most of that food, water and fuel?

It would also be justification for orbital manufacturing, if nothing else.

Byron said...

Nope. I didn't include fuel, and there's maybe 25% food and water. If you take fuel, double the numbers for a mechanized unit.

KraKon said...

And if replace all that by a single nuclear battery, spare parts and ammo only?

I'm using robots as my main invading force. The human controllers are in a ratio of 5 to 1 on the ground, 8 to one overall on the surface. That drastically reduces the logistics payload; the robots are turned into mechanized generalist labour after the invasion, the soldiers become occupational troops.

I know I'm going deeper into magitech, but my soldiers are grown in transit too. So all the ships have to do is assemble spare parts and put clothes on all the new clones. You're telling me this won't save overall mass, but it's much easier when all you have to carry is ammo, spare parts and proteins instead of the 37k tons figure in so many diverse things.

"I didn't include fuel, and there's maybe 25% food and water"

Fuel? You're carrying gasoline all the way from another solar system? I don't see water as much of a problem though as with enough energy, it is infinitely recyclable (react hydrogen fuel with onboard oxygen, boil the grey water, separate the atoms for more oxygen....)

"However, it would take 1525.5 space shuttle launches to put that in orbit, or 312.8 Saturn Vs. "

Amazing, innit, when you consider doing something 500 years in the future with technology outdated 40 years ago. How did you build that shiny interstellar invasion fleet if it took you 300 launches for the mass of a single ship?

"Also, the total supplies per soldier can quite possibly be brought down using local resources - and compared to planetbound logistics, with interplanetary invasion there's much more incentive to live off the land, so to speak."

I don't know what to say. On one hand, with enough time and energy, living off the land would be the daily occupation of invading troops as they have the technology and motivation to do so. On the other hand, there's no way 'living of the land' would support 200k troops on the move and under constant danger.

I use it in a scenario where Marines raid dome cities and take rations with them as a tax, but on large scale planetary invasions, living off the land only supplements rations bought in by ship.

Byron said...

Then the requirements drop 25%. Maybe less. I can't see drones requiring less maintainence than human-operated equipment. Fuel isn't included. Ammo will be about the same. And it makes up half of my combat numbers. Most of the savings in peacekeeping comes from ammo reductions. Plus, you'll still need logistics troops, such as robot repairmen. Either way, you can't reduce it too much.
I said that I wasn't carrying fuel. And yes, maybe my numbers are 15% to 25% too high. However, ground warfare has seen ever-increasing amounts of supplies per person over the past century. I don't think that it is unreasonable to assume that the food and water is replaced by more munitions and parts.
What do you mean, the mass of a single ship? The supplies were for the best-case occupation of a planet of 10 million. For 8.6 months. Not a single ship. And the invasion was interplanetary, not interstellar.
Troops won't live off the land during combat. They'll get water, but nothing else. They might, however, get food while on occupation duty.
Cloning like that requires a lot of magitech, and my scenario's ships aren't in transit for that long.

KraKon said...

What I was comparing to is that if you have the capacity to lift enough material to construct a ship in space, you have a much easier time lifting enough supplies for your troops with that capacity.

"Troops won't live off the land during combat. They'll get water, but nothing else."

More the contrary. They get food dropped but they get their water from the environment.

Oh sorry, you meant that.

"Cloning like that requires a lot of magitech, and my scenario's ships aren't in transit for that long."

It arised from the need to have human pilots, so I disembodied them to keep them competitive in face of drones, then I said to myself, if I can disembody brains and keep them alive, why not give them bodies and make 'em walk about on planets?

Byron said...

Not really. Given Rick's cost rule, ships will be a lot more expensive than supplies. And lots of a ship is bulk stuff that makes sense to manufacture in space. In the scenario discussed, a $1 billion warship that I built was around 300 tons empty. Supplies are far cheaper than $3 million/ton. The cloning could change the equation some, but not a lot. Most of the supplies are for operations on the ground.

BeltBandid said...

Hi I'm new.

I may be swerving off topic here because I see you all started logistics of space invasions.
But there are scenarios without defenders and attackers, like fighting for an unclaimed planet, and such, and not only that but planetary conflicts, where both sides are from the planet, like a civil war.


thing is there is a little detail i didn't see anyone mention, and this detail makes your robot armies obsolete:
Jamming, any remote control robots are thereby easily defeated by simply getting too much static in the air, since armies will want to out gun each other in jamming eq, the result will be static filled comm channels so that only communication can be made only in optical range, this makes pilots the only control source able enough to carry on warfare.

If you go by my hypothesis how would that change your theories on surface war?

Byron said...

This was discussed earlier some. (I don't blame you for not knowing. I'm not sure of all that's gone on.)
Jamming and EW are the reasons we haven't concluded that drones are inevitable. The recent discussion was merely over the logistics of drones vs. humans.
If we can't use drones extensively, you're back to people. Probably less people in things like tanks, but people still. That's pretty much all.
Fighting for an unclaimed planet will occur in space. There are no orbital defenses to prevent bombardment, and you need to move stuff to settle an unclaimed planet.
As to what a purely ground war would look like, it's sort of a purple v. green thing. We don't know enough to be sure what we'd see.

Milo said...

And so this thread falls off the main page. The end of an era...

Byron said...

So? We're still here.
BTW, I meant to put this up earlier, but I sort of forgot. An excellent description of "Space Marines" is in the AVT Nexus Journal 1. Please note, it doesn't satisfy my conditions for my scenario. (The orbital defenses are invulnerable and non-homogeneous, and the ground defenses weak.)

Byron said...

Happy Post 800!!!!
I got it this time, Milo.

Rick said...

And #801, to say welcome to another new commenter!

Wars fought entirely on one planet are more likely than planetary invasions if only because they include wars fought on Earth. But whether post industrial powers can fight each other 'seriously' without wiping each other out is an interesting problem.

«Oldest ‹Older   601 – 800 of 828   Newer› Newest»